SlideShare a Scribd company logo
1 of 51
Download to read offline
January 31, 2005 12:53       L24-ch10          Sheet number 1 Page number 424             black



                                                            CHAPTER 10
                                                         Infinite Series

             EXERCISE SET 10.1
                            1                            (−1)n−1                     2n − 1                        n2
              1. (a)                               (b)                         (c)                        (d)
                          3n−1                             3n−1                        2n                       π 1/(n+1)

              2. (a) (−r)n−1 ; (−r)n                                           (b) (−1)n+1 rn ; (−1)n rn+1

              3. (a) 2, 0, 2, 0                            (b) 1, −1, 1, −1                       (c) 2(1 + (−1)n ); 2 + 2 cos nπ

              4. (a) (2n)!                                                     (b) (2n − 1)!

                                                             n
              5. 1/3, 2/4, 3/5, 4/6, 5/7, . . . ; lim           = 1, converges
                                                     n→+∞   n+2

                                                                n2
              6. 1/3, 4/5, 9/7, 16/9, 25/11, . . . ; lim             = +∞, diverges
                                                         n→+∞ 2n + 1


              7. 2, 2, 2, 2, 2, . . .; lim 2 = 2, converges
                                     n→+∞


                         1    1    1    1
              8. ln 1, ln , ln , ln , ln , . . . ; lim ln(1/n) = −∞, diverges
                         2    3    4    5         n→+∞


                   ln 1 ln 2 ln 3 ln 4 ln 5
              9.       ,    ,    ,    ,     ,...;
                    1    2    3    4      5
                         ln n           1                               ln x
                    lim       = lim       = 0 apply L’Hˆpital’s Rule to
                                                       o                     , converges
                   n→+∞ n       n→+∞ n                                   x

             10. sin π, 2 sin(π/2), 3 sin(π/3), 4 sin(π/4), 5 sin(π/5), . . . ;
                                                sin(π/n)       (−π/n2 ) cos(π/n)
                     lim n sin(π/n) = lim                = lim                   = π, converges
                   n→+∞                    n→+∞    1/n    n→+∞     −1/n2


             11. 0, 2, 0, 2, 0, . . . ; diverges

                                                              (−1)n+1
             12. 1, −1/4, 1/9, −1/16, 1/25, . . . ; lim               = 0, converges
                                                         n→+∞   n2

             13. −1, 16/9, −54/28, 128/65, −250/126, . . . ; diverges because odd-numbered terms approach
                 −2, even-numbered terms approach 2.

                                                                n          1
             14. 1/2, 2/4, 3/8, 4/16, 5/32, . . . ; lim            = lim n      = 0, converges
                                                         n→+∞   2n  n→+∞ 2 ln 2


                                                                     1
             15. 6/2, 12/8, 20/18, 30/32, 42/50, . . . ; lim           (1 + 1/n)(1 + 2/n) = 1/2, converges
                                                             n→+∞    2

             16. π/4, π 2 /42 , π 3 /43 , π 4 /44 , π 5 /45 , . . . ; lim (π/4)n = 0, converges
                                                                n→+∞


             17. cos(3), cos(3/2), cos(1), cos(3/4), cos(3/5), . . . ; lim cos(3/n) = 1, converges
                                                                             n→+∞



                                                                       424
January 31, 2005 12:53        L24-ch10               Sheet number 2 Page number 425            black



              Exercise Set 10.1                                                                                                425


              18. 0, −1, 0, 1, 0, . . . ; diverges

                                                                                         x2
              19. e−1 , 4e−2 , 9e−3 , 16e−4 , 25e−5 , . . . ; lim x2 e−x = lim              = 0, so lim n2 e−n = 0, converges
                                                                     x→+∞           x→+∞ ex        n→+∞

                       √             √             √             √
              20. 1,       10 − 2,       18 − 3,       28 − 4,
                                               40 − 5, . . . ;
                                                    3n                                           3             3
                     lim ( n2 + 3n − n) = lim √                = lim                                       =     , converges
                    n→+∞                 n→+∞   n 2 + 3n + n    n→+∞                         1 + 3/n + 1       2

                                                                                     x
                                                                               x+3
              21. 2, (5/3)2 , (6/4)3 , (7/5)4 , (8/6)5 , . . . ; let y =                 , converges because
                                                                               x+1
                                                     x+3
                                                ln                    2x2                   n+3
                                                                                                                 n
                      lim ln y = lim                 x + 1 = lim                = 2, so lim                          = e2
                    x→+∞               x→+∞          1/x    x→+∞ (x + 1)(x + 3)        n→+∞ n + 1


              22. −1, 0, (1/3)3 , (2/4)4 , (3/5)5 , . . . ; let y = (1 − 2/x)x , converges because
                                                ln(1 − 2/x)         −2
                      lim ln y = lim                        = lim         = −2, lim (1 − 2/n)n = lim y = e−2
                    x→+∞               x→+∞         1/x      x→+∞ 1 − 2/x      n→+∞             x→+∞

                                  +∞
                       2n − 1                        2n − 1
              23.                        ; lim              = 1, converges
                         2n       n=1
                                          n→+∞         2n

                                +∞
                       n−1                   n−1
              24.                      ; lim     = 0, converges
                        n2      n=1
                                         n→+∞ n2

                                         +∞
                                   1                 (−1)n−1
              25.     (−1)n−1                  ; lim         = 0, converges
                                  3n     n=1
                                                n→+∞   3n

                                  +∞
              26. {(−1)n n}n=1 ; diverges because odd-numbered terms tend toward −∞,
                    even-numbered terms tend toward +∞.
                                         +∞
                       1   1                               1   1
              27.        −                     ; lim         −          = 0, converges
                       n n+1             n=1
                                                n→+∞       n n+1

                                  +∞
              28.    3/2n−1          ; lim 3/2n−1
                                  n=1 n→+∞
                                                            = 0, converges

                     √        √      +∞
              29.      n + 1 − n + 2 n=1 ; converges because
                          √       √                  (n + 1) − (n + 2)            −1
                     lim ( n + 1 − n + 2) = lim √              √       = lim √      √   =0
                    n→+∞                      n→+∞     n+1+ n+2         n→+∞   n+1+ n+2
                                           +∞
              30.    (−1)n+1 /3n+4            ; lim (−1)n+1 /3n+4
                                           n=1 n→+∞
                                                                            = 0, converges

                                +1 k even
              31. an =                     oscillates; there is no limit point which attracts all of the an .
                            −1 k odd
                    bn = cos n; the terms lie all over the interval [−1, 1] without any limit.

              32. (a) No, because given N > 0, all values of f (x) are greater than N provided x is close enough
                      to zero. But certainly the terms 1/n will be arbitrarily close to zero, and when so then
                      f (1/n) > N , so f (1/n) cannot converge.
                  (b) f (x) = sin(π/x). Then f = 0 when x = 1/n and f = 0 otherwise; indeed, the values of f are
                      located all over the interval [−1, 1].
January 31, 2005 12:53      L24-ch10            Sheet number 3 Page number 426                  black



             426                                                                                                            Chapter 10


                                                                       n,         n odd                         1/n,             n odd
             33. (a) 1, 2, 1, 4, 1, 6             (b)       an =                                  (c)   an =
                                                                              n
                                                                       1/2 , n even                             1/(n + 1), n even

                   (d) In Part (a) the sequence diverges, since the even terms diverge to +∞ and the odd terms
                       equal 1; in Part (b) the sequence diverges, since the odd terms diverge to +∞ and the even
                       terms tend to zero; in Part (c) lim an = 0.
                                                                    n→+∞


             34. The even terms are zero, so the odd terms must converge to zero, and this is true if and only if
                    lim bn = 0, or −1 < b < 1.
                   n→+∞

                           √                      √
                                                  n
             35.    lim    n
                               n = 1, so lim           n3 = 13 = 1
                   n→+∞                   n→+∞


                                    1            a                            1    a                        √              √
             37.    lim xn+1 =         lim xn +                    or L =       L+   , 2L2 − L2 − a = 0, L = a (we reject − a
                   n→+∞             2 n→+∞      xn                            2    L
                   because xn > 0, thus L ≥ 0.)
                                   √
             38. (a) an+1 =            6 + an
                                                   √                      √
                   (b)     lim an+1 = lim               6 + an , L =          6 + L, L2 − L − 6 = 0, (L − 3)(L + 2) = 0,
                          n→+∞             n→+∞

                          L = −2 (reject, because the terms in the sequence are positive) or L = 3; lim an = 3.
                                                                                                                  n→+∞


                            1 2 1 2 3 1   2   3   4     3 2 5
             39. (a) 1,      + , + + ,  +   +   +   = 1, , ,
                            4 4 9 9 9 16 16 16 16       4 3 8
                                 1                        1 1          1n+1
                   (c) an =         (1 + 2 + · · · + n) = 2 n(n + 1) =      , lim an = 1/2
                                 n2                      n 2           2 n n→+∞

                            1 4 1   4  9 1   4   9   16     5 14 15
             40. (a) 1,      + ,  +   + ,  +   +   +    = 1, , ,
                            8 8 27 27 27 64 64 64 64        8 27 32
                               1 2                         1 1                  1 (n + 1)(2n + 1)
                   (c) an =       (1 + 22 + · · · + n2 ) = 3 n(n + 1)(2n + 1) =                   ,
                               n3                         n 6                   6       n2
                                           1
                           lim an = lim       (1 + 1/n)(2 + 1/n) = 1/3
                          n→+∞       n→+∞ 6


                                         sin2 n       1
             41. Let an = 0, bn =               , cn = ; then an ≤ bn ≤ cn , lim an = lim cn = 0, so lim bn = 0.
                                           n          n                     n→+∞     n→+∞           n→+∞

                                                   n                  n                                             n
                                 1+n             3                                                        n/2 + n
             42. Let an = 0, bn =         , cn =    ; then (for n ≥ 2), an ≤ bn ≤                                       = cn ,
                                   2n            4                                                          2n
                    lim an = lim cn = 0, so lim bn = 0.
                   n→+∞          n→+∞                    n→+∞

                                                                                     n
             43. (a) a1 = (0.5)2 , a2 = a2 = (0.5)4 , . . . , an = (0.5)2
                                         1
                                                  n
                   (c)     lim an = lim e2             ln(0.5)
                                                                 = 0, since ln(0.5) < 0.
                          n→+∞           n→+∞

                   (d) Replace 0.5 in Part (a) with a0 ; then the sequence converges for −1 ≤ a0 ≤ 1, because if
                       a0 = ±1, then an = 1 for n ≥ 1; if a0 = 0 then an = 0 for n ≥ 1; and if 0 < |a0 | < 1 then
                                                              n−1
                       a1 = a2 > 0 and lim an = lim e2
                             0
                                                                  ln a1
                                                                        = 0 since 0 < a1 < 1. This same argument
                                                n→+∞               n→+∞
                          proves divergence to +∞ for |a| > 1 since then ln a1 > 0.
January 31, 2005 12:53         L24-ch10             Sheet number 4 Page number 427           black



              Exercise Set 10.1                                                                                                    427


              44. f (0.2) = 0.4, f (0.4) = 0.8, f (0.8) = 0.6, f (0.6) = 0.2 and then the cycle repeats, so the sequence
                  does not converge.

              45. (a)          30




                           0                                    5
                               0

                                                                            ln(2x + 3x )       2x ln 2 + 3x ln 3
                    (b) Let y = (2x + 3x )1/x , lim ln y = lim                           = lim
                                                          x→+∞         x→+∞      x        x→+∞     2x + 3x
                                                x
                                          (2/3) ln 2 + ln 3
                           = lim                            = ln 3, so lim (2n + 3n )1/n = eln 3 = 3
                                   x→+∞      (2/3)x + 1               n→+∞

                           Alternate proof: 3 = (3n )1/n < (2n +3n )1/n < (2·3n )1/n = 3·21/n . Then apply the Squeezing
                           Theorem.

              46. Let f (x) = 1/(1 + x), 0 ≤ x ≤ 1. Take ∆xk = 1/n and x∗ = k/n then
                                                                        k
                               n                          n                                  1                        1
                                        1                          1                              1
                    an =                      (1/n) =                  ∆xk so lim an =               dx = ln(1 + x)       = ln 2
                                    1 + (k/n)                   1 + x∗
                                                                     k
                                                                             n→+∞        0       1+x                  0
                           k=1                            k=1

                                          n
                            1      1        ln n                    ln n       1
              47. an =                dx =        , lim an = lim         = lim   = 0,
                           n−1   1 x       n − 1 n→+∞         n→+∞ n − 1  n→+∞ n

                                                 ln n
                      apply L’Hˆpital’s Rule to
                               o                       , converges
                                                 n−1

                                                                         an+2      an
              48. (a) If n ≥ 1, then an+2 = an+1 + an , so                    =1+      .
                                                                         an+1     an+1

                    (c) With L = lim (an+2 /an+1 ) = lim (an+1 /an ), L = 1 + 1/L, L2 − L − 1 = 0,
                                 n→+∞               n→+∞
                                √                 √
                        L = (1 ± 5)/2, so L = (1 + 5)/2 because the limit cannot be negative.

                     1     1
              49.      −0 = <                 if n > 1/
                     n     n

                    (a) 1/ = 1/0.5 = 2, N = 3                                  (b) 1/ = 1/0.1 = 10, N = 11
                    (c) 1/ = 1/0.001 = 1000, N = 1001

                      n        1
              50.        −1 =     <                   if n + 1 > 1/ , n > 1/ − 1
                     n+1      n+1

                    (a) 1/ − 1 = 1/0.25 − 1 = 3, N = 4                         (b) 1/ − 1 = 1/0.1 − 1 = 9, N = 10
                    (c) 1/ − 1 = 1/0.001 − 1 = 999, N = 1000

                               1     1
              51. (a)            −0 = <             if n > 1/ , choose any N > 1/ .
                               n     n
                                n        1
                    (b)            −1 =     <                 if n > 1/ − 1, choose any N > 1/ − 1.
                               n+1      n+1
January 31, 2005 12:53     L24-ch10        Sheet number 5 Page number 428          black



             428                                                                                              Chapter 10


             52. If |r| < 1 then    lim rn = 0; if r > 1 then    lim rn = +∞, if r < −1 then rn oscillates between
                                   n→+∞                         n→+∞
                   positive and negative values that grow in magnitude so lim rn does not exist for |r| > 1; if r = 1
                                                                            n→+∞
                   then lim 1n = 1; if r = −1 then (−1)n oscillates between −1 and 1 so lim (−1)n does not exist.
                         n→+∞                                                               n→+∞




             EXERCISE SET 10.2
                                 1  1      1
              1. an+1 − an =       − =−          < 0 for n ≥ 1, so strictly decreasing.
                                n+1 n   n(n + 1)

                                       1          1       1
              2. an+1 − an = (1 −         ) − (1 − ) =          > 0 for n ≥ 1, so strictly increasing.
                                      n+1         n    n(n + 1)

                                 n+1     n             1
              3. an+1 − an =          −       =                  > 0 for n ≥ 1, so strictly increasing.
                                2n + 3 2n + 1   (2n + 1)(2n + 3)

                                 n+1     n              1
              4. an+1 − an =          −       =−                  < 0 for n ≥ 1, so strictly decreasing.
                                4n + 3 4n − 1    (4n − 1)(4n + 3)

              5. an+1 − an = (n + 1 − 2n+1 ) − (n − 2n ) = 1 − 2n < 0 for n ≥ 1, so strictly decreasing.

              6. an+1 − an = [(n + 1) − (n + 1)2 ] − (n − n2 ) = −2n < 0 for n ≥ 1, so strictly decreasing.

                   an+1   (n + 1)/(2n + 3)   (n + 1)(2n + 1)   2n2 + 3n + 1
              7.        =                  =                 =              > 1 for n ≥ 1, so strictly increasing.
                    an       n/(2n + 1)         n(2n + 3)        2n2 + 3n

                   an+1     2n+1     1 + 2n   2 + 2n+1         1
              8.        =          ·        =          =1+          > 1 for n ≥ 1, so strictly increasing.
                    an    1 + 2n+1     2n     1 + 2n+1     1 + 2n+1

                   an+1   (n + 1)e−(n+1)
              9.        =                = (1 + 1/n)e−1 < 1 for n ≥ 1, so strictly decreasing.
                    an        ne−n

                   an+1     10n+1    (2n)!          10
             10.        =          ·       =                  < 1 for n ≥ 1, so strictly decreasing.
                    an    (2n + 2)! 10n      (2n + 2)(2n + 1)

                   an+1   (n + 1)n+1 n!   (n + 1)n
             11.        =           · n =          = (1 + 1/n)n > 1 for n ≥ 1, so strictly increasing.
                    an     (n + 1)!  n       nn
                                       2
                   an+1    5n+1   2n    5
             12.        = (n+1)2 · n = 2n+1 < 1 for n ≥ 1, so strictly decreasing.
                    an   2         5  2

             13. f (x) = x/(2x + 1), f (x) = 1/(2x + 1)2 > 0 for x ≥ 1, so strictly increasing.

             14. f (x) = 3 − 1/x, f (x) = 1/x2 > 0 for x ≥ 1, so strictly increasing.

                                                    1 + 1/x
             15. f (x) = 1/(x + ln x), f (x) = −               < 0 for x ≥ 1, so strictly decreasing.
                                                   (x + ln x)2

             16. f (x) = xe−2x , f (x) = (1 − 2x)e−2x < 0 for x ≥ 1, so strictly decreasing.

                           ln(x + 2)           1 − ln(x + 2)
             17. f (x) =             , f (x) =               < 0 for x ≥ 1, so strictly decreasing.
                             x+2                 (x + 2)2

             18. f (x) = tan−1 x, f (x) = 1/(1 + x2 ) > 0 for x ≥ 1, so strictly increasing.
January 31, 2005 12:53        L24-ch10        Sheet number 6 Page number 429          black



              Exercise Set 10.2                                                                                       429


              19. f (x) = 2x2 − 7x, f (x) = 4x − 7 > 0 for x ≥ 2, so eventually strictly increasing.

              20. f (x) = x3 − 4x2 , f (x) = 3x2 − 8x = x(3x − 8) > 0 for x ≥ 3, so eventually strictly increasing.

                                   x              10 − x2
              21. f (x) =               , f (x) = 2        < 0 for x ≥ 4, so eventually strictly decreasing.
                              x2   + 10          (x + 10)2

                                    17           x2 − 17
              22. f (x) = x +          , f (x) =         > 0 for x ≥ 5, so eventually strictly increasing.
                                    x               x2

                    an+1   (n + 1)! 3n    n+1
              23.        =         ·    =     > 1 for n ≥ 3, so eventually strictly increasing.
                     an     3n+1     n!    3

              24. f (x) = x5 e−x , f (x) = x4 (5 − x)e−x < 0 for x ≥ 6, so eventually strictly decreasing.

              25. (a) Yes: a monotone sequence is increasing or decreasing; if it is increasing, then it is increas-
                      ing and bounded above, so by Theorem 10.2.3 it converges; if decreasing, then use Theo-
                      rem 10.2.4. The limit lies in the interval [1, 2].
                  (b) Such a sequence may converge, in which case, by the argument in Part (a), its limit is ≤ 2.
                      But convergence may not happen: for example, the sequence {−n}+∞ diverges.
                                                                                          n=1


              26. The sequence {1} is monotone but not eventually strictly monotone. Let {xn }+∞ be a sequence
                                                                                                   n=1
                  that is monotone (let’s say increasing) but not eventually strictly monotone. Then there exists some
                  N0 > 0 such that for all n ≥ N0 we have xn ≤ xn+1 . Yet for any N > 0 it is not true that if n ≥ N
                  then xn < xn+1 . Since the sequence is monotone increasing we must have xn ≤ xn+1 for all n, yet
                  infinitely many times it is false that xn < xn+1 . We conclude that for some n > N, xn = xn+1 .
                  Rephrasing, we can say that for any n > 0, there exists k, say kn , such that xkn = xkn +1 . The
                  sequence has infinitely many pairs of repeated terms.

                                      |x|n+1     |x| |x|n    |x|
              27. (a) an+1 =                  =           =      an
                                     (n + 1)!   n + 1 n!    n+1
                    (b) an+1 /an = |x|/(n + 1) < 1 if n > |x| − 1.
                    (c) From Part (b) the sequence is eventually decreasing, and it is bounded below by 0, so by
                        Theorem 10.2.4 it converges.
                                                                     |x|
                    (d) If lim an = L then from Part (a), L =                L = 0.
                          n→+∞                                   lim (n + 1)
                                                                        n→+∞
                             |x|n
                    (e)  lim      = lim an = 0
                        n→+∞ n!    n→+∞


                          √        √        √
              28. (a)         2,
                              2 + 2, 2 + 2 + 2
                              √           √        √               √        √
                    (b) a1 = 2 < 2 so a2 = 2 + a1 < 2 + 2 = 2, a3 = 2 + a2 < 2 + 2 = 2, and so on
                        indefinitely.
                    (c) a2 − a2 = (2 + an ) − a2 = 2 + an − a2 = (2 − an )(1 + an )
                         n+1  n                n             n

                    (d) an > 0 and, from Part (b), an < 2 so 2 − an > 0 and 1 + an > 0 thus, from Part (c),
                          a2 − a2 > 0, an+1 − an > 0, an+1 > an ; {an } is a strictly increasing sequence.
                           n+1  n

                    (e) The sequence is increasing and has 2 as an upper bound so it must converge to a limit L,
                                              √               √
                         lim an+1 = lim         2 + an , L =    2 + L, L2 − L − 2 = 0, (L − 2)(L + 1) = 0
                          n→+∞                n→+∞
                          thus lim an = 2.
                                   n→+∞
January 31, 2005 12:53        L24-ch10          Sheet number 7 Page number 430                               black



             430                                                                                                                             Chapter 10

                                                                                               √
             29. (a) If f (x) = 1 (x + 3/x), then f (x) = (x2 − 3)/(2x2 ) and f (x) = 0 for x = 3; the minimum
                                2                   √     √                  √                           √
                     value of f (x) for x > 0 is f ( 3) = 3. Thus f (x) ≥ 3 for x > 0 and hence an ≥ 3 for
                     n ≥ 2.
                                                                                √
                 (b) an+1 − an = (3 − a2 )/(2an ) ≤ 0 for n ≥ 2 since an ≥ 3 for n ≥ 2; {an } is eventually
                                           n
                     decreasing.
                     √
                 (c)   3 is a lower bound for an so {an } converges; lim an+1 = lim 1 (an + 3/an ),
                                                                                    n→+∞ 2
                                                        √            n→+∞
                     L = 2 (L + 3/L), L − 3 = 0, L = 3.
                           1             2


             30. (a) The altitudes of the rectangles are ln k for k = 2 to n, and their bases all have length 1 so
                     the sum of their areas is ln 2 + ln 3 + · · · + ln n = ln(2 · 3 · · · n) = ln n!. The area under the
                                                                                            n                         n+1
                         curve y = ln x for x in the interval [1, n] is                         ln x dx, and                ln x dx is the area for x in
                                                                                        1                         1
                                                                                       n                                   n+1
                         the interval [1, n + 1] so, from the figure,                    ln x dx < ln n! <                        ln x dx.
                                                                                   1                                   1
                              n                            n                                          n+1
                   (b)            ln x dx = (x ln x − x)       = n ln n − n + 1 and                         ln x dx = (n + 1) ln(n + 1) − n so from
                          1                                1                                      1

                         Part (a), n ln n − n + 1 < ln n! < (n + 1) ln(n + 1) − n, en ln n−n+1 < n! < e(n+1) ln(n+1)−n ,
                                                                               nn         (n + 1)n+1
                         en ln n e1−n < n! < e(n+1) ln(n+1) e−n ,              n−1
                                                                                   < n! <
                                                                              e               en
                                                 nn
                                                       1/n         √
                                                                 (n + 1)n+1
                                                                   n
                                                                                                      1/n
                   (c) From Part (b),                          <       n! <    ,
                                                en−1                 en
                                                                     √
                              n        √
                                       n     (n + 1)1+1/n    1       n
                                                                       n!   (1 + 1/n)(n + 1)1/n
                                      < n! <              , 1−1/n <       <                     ,
                         e1−1/n                    e       e          n              e
                                                                                                    √
                                     1         1     (1 + 1/n)(n + 1)1/n  1                         n
                                                                                                      n!  1
                         but               →     and                     → as n → +∞ (why?), so lim      = .
                                  e1−1/n       e              e           e                    n→+∞ n     e

                          nn √ n      n            n               √
                                                                   n
             31. n! >     n−1
                              , n! > 1−1/n , lim 1−1/n = +∞ so lim   n! = +∞.
                         e          e       n→+∞ e            n→+∞




             EXERCISE SET 10.3
                                                       62        312      2 − 2(1/5)n  5 5
              1. (a) s1 = 2, s2 = 12/5, s3 =              , s4 =     sn =             = − (1/5)n ,
                                                       25        125        1 − 1/5    2 2
                                           5
                          lim sn =           , converges
                         n→+∞              2
                             1       3      7       15      (1/4) − (1/4)2n    1 1
                   (b) s1 =    , s2 = , s3 = , s4 =    sn =                 = − + (2n ),
                             4       4      4       4            1−2           4 4
                          lim sn = +∞, diverges
                         n→+∞

                               1            1     1        1      1        3       1
                   (c)                  =     −      , s1 = , s2 = , s3 =    , s4 = ;
                         (k + 1)(k + 2)   k+1 k+2          6      4       10       3
                              1      1             1
                         sn = −         , lim sn = , converges
                              2 n + 2 n→+∞         2
January 31, 2005 12:53           L24-ch10                  Sheet number 8 Page number 431                  black



              Exercise Set 10.3                                                                                                                 431


               2. (a) s1 = 1/4, s2 = 5/16, s3 = 21/64, s4 = 85/256
                                                                           n−1                                         n
                                     1                1               1                1 1 − (1/4)n   1            1                      1
                             sn =            1+         + ··· +                    =                =      1−              ;   lim sn =
                                     4                4               4                4 1 − 1/4      3            4           n→+∞       3

                                                                                       4n − 1
                    (b) s1 = 1, s2 = 5, s3 = 21, s4 = 85; sn =                                , diverges
                                                                                          3
                    (c) s1 = 1/20, s2 = 1/12, s3 = 3/28, s4 = 1/8;
                                         n
                                                  1   1                         1   1
                             sn =                   −                     =       −    , lim sn = 1/4
                                                 k+3 k+4                        4 n + 4 n→+∞
                                     k=1

                                                                                     1
               3. geometric, a = 1, r = −3/4, sum =                                        = 4/7
                                                                                1 − (−3/4)

                                                                                   (2/3)3
               4. geometric, a = (2/3)3 , r = 2/3, sum =                                   = 8/9
                                                                                   1 − 2/3

                                                                                   7
               5. geometric, a = 7, r = −1/6, sum =                                     =6
                                                                                1 + 1/6

               6. geometric, r = −3/2, diverges
                              n
                                      1   1                           1   1
               7. sn =                  −                         =     −    , lim sn = 1/3
                                     k+2 k+3                          3 n + 3 n→+∞
                             k=1

                              n
                                     1     1                    1   1
               8. sn =                  − k+1               =     −   , lim sn = 1/2
                                     2k  2                      2 2n+1 n→+∞
                             k=1

                              n
                                      1/3    1/3                          1   1/3
               9. sn =                     −                          =     −     , lim sn = 1/6
                                     3k − 1 3k + 2                        6 3n + 2 n→+∞
                             k=1

                             n+1                                          n+1           n+1
                                    1/2   1/2   1                                1              1
              10. sn =                  −     =                                     −
                                    k−1 k+1     2                               k−1            k+1
                             k=2                                          k=2            k=2

                                   n+1                 n+3
                             1            1                   1    1   1  1   1                                                3
                         =                   −                   =   1+ −   −    ;                              lim sn =
                             2           k−1                 k−1   2   2 n+1 n+2                              n→+∞             4
                                   k=2                 k=4

                     ∞                       ∞
                           1
              11.             =                  1/k, the harmonic series, so the series diverges.
                          k−2
                    k=3                  k=1


                                                                                        (e/π)4      e4
              12. geometric, a = (e/π)4 , r = e/π < 1, sum =                                    = 3
                                                                                        1 − e/π  π (π − e)
                     ∞                   ∞                  k−1
                          4k+2                         4                                                       64
              13.              =                 64               ; geometric, a = 64, r = 4/7, sum =                = 448/3
                          7k−1                         7                                                     1 − 4/7
                    k=1                  k=1


              14. geometric, a = 125, r = 125/7, diverges

              15. (a) Exercise 5                              (b) Exercise 3                    (c) Exercise 7                 (d) Exercise 9
January 31, 2005 12:53         L24-ch10             Sheet number 9 Page number 432                              black



             432                                                                                                                                    Chapter 10


             16. (a) Exercise 10                            (b) Exercise 6                           (c) Exercise 4                   (d) Exercise 8

                                                                                    0.4
             17. 0.4444 · · · = 0.4 + 0.04 + 0.004 + · · · =                              = 4/9
                                                                                  1 − 0.1
                                                                                    0.9
             18. 0.9999 · · · = 0.9 + 0.09 + 0.009 + · · · =                              =1
                                                                                  1 − 0.1
                                                                                                           0.37
             19. 5.373737 · · · = 5 + 0.37 + 0.0037 + 0.000037 + · · · = 5 +                                      = 5 + 37/99 = 532/99
                                                                                                         1 − 0.01
                                                                                                                                      0.00014    44663
             20. 0.451141414 · · · = 0.451 + 0.00014 + 0.0000014 + 0.000000014 + · · · = 0.451 +                                               =
                                                                                                                                      1 − 0.01   99000

             21. 0.a1 a2 · · · an 9999 · · · = 0.a1 a2 · · · an + 0.9 (10−n ) + 0.09 (10−n ) + · · ·
                                                                                0.9 (10−n )
                                                = 0.a1 a2 · · · an +                        = 0.a1 a2 · · · an + 10−n
                                                                                  1 − 0.1
                                                = 0.a1 a2 · · · (an + 1) = 0.a1 a2 · · · (an + 1) 0000 · · ·

             22. The series converges to 1/(1 − x) only if −1 < x < 1.

                                    3           3 3          3 3 3
             23. d = 10 + 2 ·         · 10 + 2 · · · 10 + 2 · · · · 10 + · · ·
                                    4           4 4          4 4 4
                                                                2                 3
                                      3                 3                   3                          20(3/4)
                     = 10 + 20              + 20                    + 20              + · · · = 10 +           = 10 + 60 = 70 meters
                                      4                 4                   4                          1 − 3/4
                                                3                   3                       3                           2                   n
                                            1               1                          1                      1     1                   1
             24. volume = 13 +                      +                   + ··· +                 + ··· = 1 +     +           + ··· +             + ···
                                            2               4                         2n                      8     8                   8
                                       1
                            =                = 8/7
                                   1 − (1/8)

                                1     2      3            n                                          1 2 3     n                       1
             25. (a) sn = ln      + ln + ln + · · · + ln     = ln                                     · · ···                 = ln        = − ln(n + 1),
                                2     3      4           n+1                                         2 3 4    n+1                     n+1
                           lim sn = −∞, series diverges.
                          n→+∞

                                                     k2 − 1      (k − 1)(k + 1)      k−1      k+1      k−1       k
                   (b) ln(1 − 1/k 2 ) = ln                  = ln                = ln     + ln     = ln     − ln     ,
                                                       k2              k2             k        k        k       k+1
                                   n+1
                                               k−1       k
                          sn =            ln       − ln
                                                k       k+1
                                   k=2

                                          1      2                      2      3                  3      4                    n       n+1
                               =    ln      − ln        + ln              − ln         + ln         − ln      + · · · + ln       − ln
                                          2      3                      3      4                  4      5                   n+1      n+2
                                      1      n+1               1
                               = ln     − ln      , lim sn = ln = − ln 2
                                      2      n + 2 n→+∞        2
                           ∞
                                                                                               1       1
             26. (a)            (−1)k xk = 1 − x + x2 − x3 + · · · =                                =     if | − x| < 1, |x| < 1, −1 < x < 1.
                                                                                           1 − (−x)   1+x
                          k=0
                           ∞
                                                                                                         1         1
                   (b)          (x − 3)k = 1 + (x − 3) + (x − 3)2 + · · · =                                     =     if |x − 3| < 1, 2 < x < 4.
                                                                                                    1 − (x − 3)   4−x
                          k=0
                           ∞
                                                                                               1           1
                   (c)          (−1)k x2k = 1−x2 +x4 −x6 +· · · =                                     =        if |−x2 | < 1, |x| < 1, −1 < x < 1.
                                                                                           1 − (−x2 )   1 + x2
                          k=0
January 31, 2005 12:53           L24-ch10           Sheet number 10 Page number 433                           black



              Exercise Set 10.3                                                                                                          433


              27. (a) Geometric series, a = x, r = −x2 . Converges for | − x2 | < 1, |x| < 1;
                              x            x
                      S=             =         .
                         1 − (−x  2)    1 + x2
                    (b) Geometric series, a = 1/x2 , r = 2/x. Converges for |2/x| < 1, |x| > 2;
                             1/x2          1
                        S=          = 2         .
                           1 − 2/x     x − 2x
                    (c) Geometric series, a = e−x , r = e−x . Converges for |e−x | < 1, e−x < 1, ex > 1, x > 0;
                             e−x          1
                        S=       −x
                                    = x      .
                           1−e         e −1

                                                      1                         1
              28. Geometric series, a = sin x, r = − sin x. Converges for | − sin x| < 1, | sin x| < 2,
                                                      2                         2
                                                           sin x     2 sin x
                  so converges for all values of x. S =           =           .
                                                            1       2 + sin x
                                                        1 + sin x
                                                            2

                            1     1      1    1  1      1  1      1    1  1      1   1  1
              29. a2 =        a1 + , a3 = a2 + = 2 a1 + 2 + , a4 = a3 + = 3 a1 + 3 + 2 + ,
                            2     2      2    2 2      2   2      2    2 2      2   2   2
                            1     1  1      1   1   1  1                1        1     1           1
                    a5 =      a4 + = 4 a1 + 4 + 3 + 2 + , . . . , an = n−1 a1 + n−1 + n−2 + · · · + ,
                            2     2 2      2   2   2   2              2        2     2             2
                                                              ∞            n
                                                a1                    1                1/2
                     lim an = lim                         +                    =0+           =1
                    n→+∞                n→+∞ 2n−1
                                                              n=1
                                                                      2              1 − 1/2

                    √         √  √      √
                        k+1− k     k+1− k   1    1
              30.       √       = √ √     =√ −√     ,
                          k 2+k     k k+1    k  k+1
                             n
                                       1    1                          1  1             1  1                    1  1
                    sn =              √ −√                    =       √ −√        +    √ −√              +     √ −√
                            k=1
                                        k  k+1                          1  2             2  3                    3  4
                                          1    1                                1
                            +··· +       √ −√                       =1− √           ; lim sn = 1
                                           n  n+1                              n + 1 n→+∞

              31. sn = (1 − 1/3) + (1/2 − 1/4) + (1/3 − 1/5) + (1/4 − 1/6) + · · · + [1/n − 1/(n + 2)]
                        = (1 + 1/2 + 1/3 + · · · + 1/n) − (1/3 + 1/4 + 1/5 + · · · + 1/(n + 2))
                        = 3/2 − 1/(n + 1) − 1/(n + 2), lim sn = 3/2
                                                                          n→+∞


                             n                      n                                  n          n
                                     1                     1/2    1/2   1                   1            1
              32. sn =                     =                   −      =                       −
                                  k(k + 2)                  k    k+2    2                   k           k+2
                            k=1                     k=1                               k=1         k=1
                                  n           n+2
                            1           1           1         1   1  1   1                                            3
                        =                 −               =     1+ −   −    ;                           lim sn =
                            2           k           k         2   2 n+1 n+2                           n→+∞            4
                                  k=1         k=3


                             n                                    n                                     n                 n
                                         1                                 1/2    1/2     1                     1                 1
              33. sn =                             =                            −       =                            −
                                  (2k − 1)(2k + 1)                        2k − 1 2k + 1   2                   2k − 1            2k + 1
                            k=1                                k=1                                      k=1               k=1
                                  n                  n+1
                            1             1                  1      1      1                                      1
                        =                      −                  =   1−        ;                  lim sn =
                            2           2k − 1             2k − 1   2    2n + 1                   n→+∞            2
                                  k=1                k=2
January 31, 2005 12:53        L24-ch10      Sheet number 11 Page number 434                          black



             434                                                                                                                              Chapter 10


             34. P0 P1 = a sin θ,                                                                        a sin u cos u
                                                                                                 P1
                 P1 P2 = a sin θ cos θ,
                 P2 P3 = a sin θ cos2 θ,                                                                      P3         a sin u cos2 u
                                                                             a sin u
                 P3 P4 = a sin θ cos3 θ, . . .                                              u                             a sin u cos3 u
                                                                                                     u
                 (see figure)
                 Each sum is a geometric series.                                                                            u
                                                                                                                                          P
                                                                                 P0             P2          P4
                                                                                                            a

                                                                                                                              a sin θ
                   (a) P0 P1 + P1 P2 + P2 P3 + · · · = a sin θ + a sin θ cos θ + a sin θ cos2 θ + · · · =
                                                                                                                             1 − cos θ
                   (b) P0 P1 + P2 P3 + P4 P5 + · · · = a sin θ + a sin θ cos2 θ + a sin θ cos4 θ + · · ·
                                                          a sin θ      a sin θ
                                                     =             =           = a csc θ
                                                       1 − cos2 θ      sin2 θ
                   (c) P1 P2 + P3 P4 + P5 P6 + · · · = a sin θ cos θ + a sin θ cos3 θ + · · ·
                                                       a sin θ cos θ    a sin θ cos θ
                                                     =               =                = a cot θ
                                                        1 − cos2 θ          sin2 θ
                                      θ θ θ  θ             θ/2
             35. By inspection,        − + −   + ··· =            = θ/3
                                      2 4 8 16         1 − (−1/2)
                                                                           1
             36. A1 + A2 + A3 + · · · = 1 + 1/2 + 1/4 + · · · =                  =2
                                                                       1 − (1/2)

                           2k A       2k B       2k 3k+1 − 2k+1 A + 2k 3k − 2k B
             37. (b)              + k+1        =
                         3k − 2 k  3    − 2k+1        (3k − 2k ) (3k+1 − 2k+1 )
                                                          3 · 6k − 2 · 22k A + 6k − 22k B   (3A + B)6k − (2A + B)22k
                                                   =                                      =
                                                               (3k − 2k ) (3k+1 − 2k+1 )      (3k − 2k ) (3k+1 − 2k+1 )
                         so 3A + B = 1 and 2A + B = 0, A = 1 and B = −2.
                                 n                                     n
                                         2k        2k+1                                                               2k
                   (c) sn =                    − k+1        =               (ak − ak+1 ) where ak =                         .
                                      3k − 2 k  3    − 2k+1                                                        3k − 2 k
                                k=1                                   k=1
                         But sn = (a1 − a2 ) + (a2 − a3 ) + (a3 − a4 ) + · · · + (an − an+1 ) which is a telescoping sum,
                                                      2n+1                         (2/3)n+1
                         sn = a1 − an+1 = 2 −                 , lim sn = lim 2 −              = 2.
                                                   3n+1 − 2n+1 n→+∞     n→+∞     1 − (2/3)n+1
                                                                                                          ∞
                                                                                                               1      1      1
             38. (a)     geometric; 18/5           (b)     geometric; diverges              (c)                           −                      = 1/2
                                                                                                               2    2k − 1 2k + 1
                                                                                                         k=1




             EXERCISE SET 10.4
                          ∞                              ∞                                      ∞
                               1      1/2                      1      1/4                                 1    1
              1. (a)              =         = 1;                  =         = 1/3;                           + k           = 1 + 1/3 = 4/3
                               2k   1 − 1/2                    4k   1 − 1/4                               2k  4
                         k=1                             k=1                                    k=1

                          ∞                                  ∞
                               1       1/5                          1
                   (b)             =         = 1/4;                       =1           (Example 5, Section 10 .4);
                               5 k   1 − 1/5                     k(k + 1)
                         k=1                               k=1
                          ∞
                                1       1
                                   −          = 1/4 − 1 = −3/4
                                5k   k(k + 1)
                         k=1
January 31, 2005 12:53         L24-ch10               Sheet number 12 Page number 435                         black



              Exercise Set 10.4                                                                                                                    435

                              ∞                                                                ∞
                                       1                                                               7       7/10
               2. (a)                      = 3/4 (Exercise 10, Section 10.4);                              =          = 7/9;
                                    k2 − 1                                                           10k−1   1 − 1/10
                           k=2                                                                 k=2
                                    ∞
                                              1     7
                           so                    −      = 3/4 − 7/9 = −1/36
                                          k 2 − 1 10k−1
                                   k=2
                                                                             ∞
                                                                                                        9/7
                    (b) with a = 9/7, r = 3/7, geometric,                         7−k 3k+1 =                   = 9/4;
                                                                                                     1 − (3/7)
                                                                            k=1
                                                                             ∞
                                                                                  2k+1      4/5
                           with a = 4/5, r = 2/5, geometric,                           =           = 4/3;
                                                                                   5k    1 − (2/5)
                                                                            k=1
                              ∞                           k+1
                                                      2
                                     7−k 3k+1 −                  = 9/4 − 4/3 = 11/12
                                                          5k
                           k=1


               3. (a) p = 3, converges                         (b) p = 1/2, diverges           (c) p = 1, diverges               (d) p = 2/3, diverges

               4. (a) p = 4/3, converges                       (b) p = 1/4, diverges           (c) p = 5/3, converges            (d) p = π, converges
                                                                                                                          k
                               k2 + k + 3  1                                                                          1
               5. (a)         lim  2+1
                                          = ; the series diverges.                             (b)      lim     1+            = e; the series diverges.
                           k→+∞ 2k         2                                                           k→+∞           k
                                                                                                               1
                    (c)            lim cos kπ does not exist;                                  (d)      lim       = 0; no information
                              k→+∞                                                                     k→+∞    k!
                              the series diverges.
                                         k
               6. (a)         lim           = 0; no information                                (b)      lim ln k = +∞; the series diverges.
                           k→+∞          ek                                                            k→+∞
                                                                                                                 √
                              1                                                                                   k
                    (c)  lim √ = 0; no information                                             (d)      lim √        = 1; the series diverges.
                        k→+∞   k                                                                       k→+∞      k+3
                                   +∞
                                           1          1
               7. (a)                           = lim   ln(5x + 2)                = +∞, the series diverges by the Integral Test.
                               1         5x + 2   →+∞ 5                       1

                                   +∞
                                            1             1                               1
                    (b)                        2
                                                 dx = lim   tan−1 3x                  =     π/2 − tan−1 3 ,
                               1         1 + 9x       →+∞ 3                       1       3
                           the series converges by the Integral Test.
                                   +∞
                                          x            1
               8. (a)                       2
                                              dx = lim   ln(1 + x2 )                  = +∞, the series diverges by the Integral Test.
                               1         1+x       →+∞ 2                          1

                                   +∞                            √                                 √
                    (b)                 (4 + 2x)−3/2 dx = lim −1/ 4 + 2x                       = 1/ 6,
                               1                                   →+∞                     1
                           the series converges by the Integral Test.
                     ∞                    ∞
                           1                   1
               9.             =                  , diverges because the harmonic series diverges.
                          k+6                  k
                    k=1                  k=7

                     ∞               ∞
                           3              3       1
              10.            =                      , diverges because the harmonic series diverges.
                          5k              5       k
                    k=1             k=1

                     ∞                        ∞
                               1        1
              11.         √        =   √ , diverges because the p-series with p = 1/2 ≤ 1 diverges.
                    k=1
                              k + 5 k=6 k
January 31, 2005 12:53          L24-ch10           Sheet number 13 Page number 436                       black



             436                                                                                                                   Chapter 10


                                1
             12.     lim                = 1, the series diverges because lim uk = 1 = 0.
                   k→+∞       e1/k                                                  k→+∞


                         +∞
                                                            3
             13.              (2x − 1)−1/3 dx = lim           (2x − 1)2/3               = +∞, the series diverges by the Integral Test.
                     1                                →+∞   4                       1

                                                                      +∞
                   ln x                                                       ln x       1
             14.        is decreasing for x ≥ e, and                               = lim (ln x)2            = +∞,
                    x                                             3            x     →+∞ 2              3
                   so the series diverges by the Integral Test.

                                  k              1
             15.     lim               = lim           = +∞, the series diverges because lim uk = 0.
                   k→+∞       ln(k + 1) k→+∞ 1/(k + 1)                                  k→+∞


                         +∞
                                             1
                              xe−x dx = lim − e−x                 = e−1 /2, the series converges by the Integral Test.
                                  2              2
             16.
                     1                  →+∞  2                1


             17.     lim (1 + 1/k)−k = 1/e = 0, the series diverges.
                   k→+∞


                        k2 + 1
             18.     lim        = 1 = 0, the series diverges.
                   k→+∞ k 2 + 3


                         +∞
                              tan−1 x          1                          2
             19.                   2
                                      dx = lim   tan−1 x                          = 3π 2 /32, the series converges by the Integral Test, since
                     1         1+x         →+∞ 2                              1

                    d tan−1 x   1 − 2x tan−1 x
                              =                < 0 for x ≥ 1.
                   dx 1 + x2       (1 + x2 )2

                         +∞
                                    1
             20.              √              dx = lim sinh−1 x            = +∞, the series diverges by the Integral Test.
                     1            x2    +1        →+∞                 1


             21.     lim k 2 sin2 (1/k) = 1 = 0, the series diverges.
                   k→+∞


                         +∞
                                               1
                              x2 e−x dx = lim − e−x                   = e−1 /3,
                                    3              3
             22.
                     1                    →+∞  3                  1

                   the series converges by the Integral Test (x2 e−x is decreasing for x ≥ 1).
                                                                                          3




                     ∞
             23. 7         k −1.01 , p-series with p > 1, converges
                     k=5

                         +∞
             24.              sech2 x dx = lim tanh x             = 1 − tanh(1), the series converges by the Integral Test.
                     1                          →+∞           1

                                                                                        +∞
                      1                                                                       dx
             25.          p
                            is decreasing for x ≥ ep , so use the Integral Test with                 to get
                   x(ln x)                                                            ep   x(ln x)p
                                                                                 
                                                                        1−p
                                                                                  +∞
                                                                                           if p < 1
                                                                  (ln x)
                    lim ln(ln x)     = +∞ if p = 1,         lim                =      1−p
                    →+∞           ep                        →+∞     1 − p ep  p           if p > 1
                                                                                     p−1
                   Thus the series converges for p > 1.
January 31, 2005 12:53       L24-ch10               Sheet number 14 Page number 437                         black



              Exercise Set 10.4                                                                                                                        437


              26. If p > 0 set g(x) = x(ln x)[ln(ln x)]p , g (x) = (ln(ln x))p−1 [(1 + ln x) ln(ln x) + p], and, for x > ee ,
                                                                                                                                 +∞
                                                                                                                                               dx
                    g (x) > 0, thus 1/g(x) is decreasing for x > ee ; use the Integral Test with
                                                                                                                                ee      x(ln x)[ln(ln x)]p
                    to get                                                                                  
                                                                                              1−p
                                                                                                             +∞
                                                                                                                           if p < 1,
                                                                      [ln(ln x)]
                      lim ln[ln(ln x)]             = +∞ if p = 1, lim                                     =
                     →+∞                      ee                  →+∞     1−p                        ee      1
                                                                                                                           if p > 1
                                                                                                              p−1
                                                                                                                                      [ln(ln x)]p      1
                    Thus the series converges for p > 1 and diverges for 0 < p ≤ 1. If p ≤ 0 then                                                 ≥
                                                                                                                                         x ln x     x ln x
                    for x > ee so the series diverges.

              27. Suppose Σ(uk + vk ) converges; then so does Σ[(uk + vk ) − uk ], but Σ[(uk + vk ) − uk ] = Σvk , so
                  Σvk converges which contradicts the assumption that Σvk diverges. Suppose Σ(uk − vk ) converges;
                  then so does Σ[uk − (uk − vk )] = Σvk which leads to the same contradiction as before.

              28. Let uk = 2/k and vk = 1/k; then both Σ(uk + vk ) and Σ(uk − vk ) diverge; let uk = 1/k and
                  vk = −1/k then Σ(uk + vk ) converges; let uk = vk = 1/k then Σ(uk − vk ) converges.
                                                        ∞                                    ∞
              29. (a) diverges because                      (2/3)k−1 converges and               1/k diverges.
                                                     k=1                                  k=1
                                                        ∞                                    ∞
                    (b) diverges because                    1/(3k + 2) diverges and                1/k 3/2 converges.
                                                     k=1                                     k=1

                                                                  ∞                                        ∞
                                                                          1                                      1
              30. (a) converges because both                                    (Exercise 25) and                   converge.
                                                                       k(ln k)2                                  k2
                                                               k=2                                         k=2
                                                      +∞                                                                              +∞
                                                                                                                                               1
                                                            ke−k converges (Integral Test), and, by Exercise 25,
                                                                   2
                    (b) diverges, because                                                                                                          diverges
                                                                                                                                            k ln k
                                                      k=2                                                                             k=2

                             ∞             ∞                                                          ∞
                                   1               1                                                       1         1
              31. (a) 3               −               = π 2 /2 − π 4 /90                     (b)               − 1 − 2 = π 2 /6 − 5/4
                                   k2              k4                                                      k 2      2
                             k=1          k=1                                                        k=1
                             ∞                      ∞
                                    1                    1
                    (c)                   =                 = π 4 /90
                                 (k − 1)4                k4
                          k=2                      k=1

                                    ∞                          n                                 ∞
              32. (a) If S =              uk and sn =                  uk , then S − sn =             uk . Interpret uk , k = n + 1, n + 2, . . ., as
                                    k=1                       k=1                            k=n+1
                          the areas of inscribed or circumscribed rectangles with height uk and base of length one for
                          the curve y = f (x) to obtain the result.
                                          n
                    (b) Add sn =                uk to each term in the conclusion of Part (a) to get the desired result:
                                          k=1
                                    +∞                      +∞                    +∞
                          sn +            f (x) dx <              uk < sn +            f (x) dx
                                   n+1                      k=1                  n


                                                                                            +∞                         +∞
                                                                            1                                      1            1
              33. (a) In Exercise 32 above let f (x) =                         . Then              f (x) dx = −             =     ;
                                                                            x2           n                         x   n        n
                          use this result and the same result with n + 1 replacing n to obtain the desired result.
January 31, 2005 12:53            L24-ch10       Sheet number 15 Page number 438                       black



             438                                                                                                                Chapter 10


                                                                                  1  1          1
                   (b) s3 = 1 + 1/4 + 1/9 = 49/36; 58/36 = s3 +                     < π 2 < s3 + = 61/36
                                                                                  4  6          3
                                      1 2
                   (d) 1/11 <           π − s10 < 1/10
                                      6

             34. Apply Exercise 32 in each case:
                                                      +∞                                             ∞
                                   1                                       1           1                     1              1
                   (a) f (x) =           ,                  f (x) dx =           , so    <                         − s10 <
                               (2x + 1)2             n                 2(2n + 1)      46                 (2k + 1)2         42
                                                                                                   k=1
                                                  +∞
                                         1                              π
                   (b) f (x) =               ,           f (x) dx =       − tan−1 (n), so
                                      k2 + 1     n                      2
                                                     ∞
                                                                1
                            π/2 − tan−1 (11) <                     − s10 < π/2 − tan−1 (10)
                                                           k2   +1
                                                     k=1
                                             +∞                                                  ∞
                                      x                                                                k
                   (c) f (x) =           ,        f (x) dx = (n + 1)e−n , so 12e−11 <                     − s10 < 11e−10
                                      ex     n                                                         ek
                                                                                                 k=1

                               +∞
                                     1       1
             35. (a)                    dx = 2 ; use Exercise 32(b) sw
                              n      x3     2n
                              1        1
                   (b)           −           < 0.01 for n = 5.
                             2n2   2(n + 1)2
                   (c) From Part (a) with n = 5 obtain 1.200 < S < 1.206, so S ≈ 1.203.

                               +∞
                                     1       1                     1        1
             36. (a)                    dx = 3 ; choose n so that     −           < 0.005, n = 4; S ≈ 1.08
                              n      x4     3n                    3n3   3(n + 1)3

                                                   n                                 n+1
                                        1            1                                     1
             37. (a) Let F (x) =          , then       dx = ln n and                         dx = ln(n + 1), u1 = 1 so
                                        x         1 x                            1         x
                            ln(n + 1) < sn < 1 + ln n.
                   (b) ln(1, 000, 001) < s1,000,000 < 1 + ln(1, 000, 000), 13 < s1,000,000 < 15
                   (c) s109 < 1 + ln 109 = 1 + 9 ln 10 < 22
                   (d) sn > ln(n + 1) ≥ 100, n ≥ e100 − 1 ≈ 2.688 × 1043 ; n = 2.69 × 1043

             38. p-series with p = ln a; convergence for p > 1, a > e

             39. x2 e−x is decreasing and positive for x > 2 so the Integral Test applies:
                        ∞                                           ∞
                            x2 e−x dx = −(x2 + 2x + 2)e−x               = 5e−1 so the series converges.
                    1                                               1


             40. (a) f (x) = 1/(x3 + 1) is decreasing and continuous on the interval [1, +∞], so the Integral Test
                     applies.

                   (c)       n         10           20              30           40             50
                             sn     0.681980     0.685314        0.685966     0.686199       0.686307
                             n         60           70              80           90             100
                             sn     0.686367     0.686403        0.686426     0.686442       0.686454
January 31, 2005 12:53      L24-ch10           Sheet number 16 Page number 439                  black



              Exercise Set 10.5                                                                                                 439

                                                               √                   √
                                          +∞
                                                      1          3    1   n3 + 1     3              2n − 1
                    (e) Set g(n) =                        dx =     π + ln        −     tan−1         √     ; for n ≥ 13,
                                         n         x3 + 1       6     6 (n + 1)3    3                  3
                           g(n) − g(n + 1) ≤ 0.0005; s13          + (g(13) + g(14))/2 ≈ 0.6865, so the sum ≈ 0.6865 to three
                           decimal places.



              EXERCISE SET 10.5
                                                                ∞
                               1        1       1                      1
               1. (a)               ≤ 2       = 2,                         converges
                           5k 2 − k  5k − k 2  4k                     4k 2
                                                               k=1
                                             ∞
                              3     3
                    (b)            > ,             3/k diverges
                           k − 1/4  k
                                             k=1

                                                    ∞                                                      ∞
                           k+1     k  1                                                        2    2            2
               2. (a)        2−k
                                 > 2 = ,                  1/k diverges                (b)     4+k
                                                                                                  < 4,              converges
                           k      k   k                                                     k      k             k4
                                                    k=2                                                    k=1

                                             ∞                                                             ∞
                              1     1              1                                        5 sin2 k  5          5
               3. (a)             < k,                converges                       (b)            < ,            converges
                           3k + 5  3               3k                                          k!     k!         k!
                                             k=1                                                           k=1

                                                     ∞
                           ln k  1                         1
               4. (a)           > for k ≥ 3,                 diverges
                            k    k                         k
                                                     k=1
                                                              ∞
                              k        k     1                     1
                    (b)             > 3/2 = √ ;                   √ diverges
                        k 3/2 − 1/2  k        k               k=1
                                                                    k

                                                                  ∞
                                                                                          4k 7 − 2k 6 + 6k 5
               5. compare with the convergent series                    1/k 5 , ρ = lim                      = 1/2, converges
                                                                                     k→+∞   8k 7 + k − 8
                                                                  k=1

                                                                  ∞
                                                                                          k
               6. compare with the divergent series                   1/k, ρ = lim             = 1/9, diverges
                                                                                   k→+∞ 9k + 6
                                                                k=1

                                                                  ∞
                                                                                            3k
               7. compare with the convergent series                    5/3k , ρ = lim           = 1, converges
                                                                                     k→+∞ 3k + 1
                                                                  k=1

                                                                  ∞
                                                                                              k 2 (k + 3)
               8. compare with the divergent series                   1/k, ρ = lim                            = 1, diverges
                                                                                   k→+∞ (k + 1)(k + 2)(k + 5)
                                                                k=1

                                                                  ∞
                                                                        1
               9. compare with the divergent series                           ,
                                                                k=1
                                                                      k 2/3
                                       k 2/3                 1
                    ρ = lim                      = lim              = 1/2, diverges
                          k→+∞ (8k 2   − 3k) 1/3  k→+∞ (8 − 3/k)1/3

                                                                  ∞
              10. compare with the convergent series                    1/k 17 ,
                                                                  k=1

                               k 17               1
                    ρ = lim         17
                                       = lim             = 1/217 , converges
                       k→+∞ (2k + 3)    k→+∞ (2 + 3/k)17
January 31, 2005 12:53       L24-ch10        Sheet number 17 Page number 440               black



             440                                                                                                        Chapter 10


                              3k+1 /(k + 1)!         3
             11. ρ = lim                     = lim       = 0, the series converges
                         k→+∞     3k /k!      k→+∞ k + 1


                              4k+1 /(k + 1)2         4k 2
             12. ρ = lim                     = lim          = 4, the series diverges
                         k→+∞     4k /k 2     k→+∞ (k + 1)2


                                k
             13. ρ = lim            = 1, the result is inconclusive
                         k→+∞ k + 1


                              (k + 1)(1/2)k+1       k+1
             14. ρ = lim                      = lim     = 1/2, the series converges
                         k→+∞     k(1/2)k      k→+∞ 2k


                              (k + 1)!/(k + 1)3          k3
             15. ρ = lim                        = lim          = +∞, the series diverges
                         k→+∞       k!/k 3       k→+∞ (k + 1)2


                              (k + 1)/[(k + 1)2 + 1]       (k + 1)(k 2 + 1)
             16. ρ = lim                             = lim                  = 1, the result is inconclusive.
                         k→+∞      k/(k 2 + 1)        k→+∞ k(k 2 + 2k + 2)


                                3k + 2
             17. ρ = lim               = 3/2, the series diverges
                         k→+∞   2k − 1

             18. ρ = lim k/100 = +∞, the series diverges
                         k→+∞


                             k 1/k
             19. ρ = lim           = 1/5, the series converges
                         k→+∞ 5


             20. ρ = lim (1 − e−k ) = 1, the result is inconclusive
                         k→+∞


             21. Ratio Test, ρ = lim 7/(k + 1) = 0, converges
                                     k→+∞

                                                                                      ∞
             22. Limit Comparison Test, compare with the divergent series                  1/k
                                                                                     k=1

                                          (k + 1)2
             23. Ratio Test, ρ = lim               = 1/5, converges
                                     k→+∞   5k 2

             24. Ratio Test, ρ = lim (10/3)(k + 1) = +∞, diverges
                                     k→+∞


             25. Ratio Test, ρ = lim e−1 (k + 1)50 /k 50 = e−1 < 1, converges
                                     k→+∞

                                                                                      ∞
             26. Limit Comparison Test, compare with the divergent series                  1/k
                                                                                     k=1

                                                                                           ∞
                                                                                                                        k3
             27. Limit Comparison Test, compare with the convergent series                      1/k 5/2 , ρ =    lim         = 1,
                                                                                                                k→+∞ k 3 + 1
                 converges                                                                k=1

                                      ∞                                     ∞
                     4      4                4                                       4
             28.       kk
                          < k ,              kk
                                                converges (Ratio Test) so                 converges by the Comparison Test
                   2+3     3 k             3                                      2 + k3k
                                     k=1                                    k=1
Chapter 10
Chapter 10
Chapter 10
Chapter 10
Chapter 10
Chapter 10
Chapter 10
Chapter 10
Chapter 10
Chapter 10
Chapter 10
Chapter 10
Chapter 10
Chapter 10
Chapter 10
Chapter 10
Chapter 10
Chapter 10
Chapter 10
Chapter 10
Chapter 10
Chapter 10
Chapter 10
Chapter 10
Chapter 10
Chapter 10
Chapter 10
Chapter 10
Chapter 10
Chapter 10
Chapter 10
Chapter 10
Chapter 10
Chapter 10

More Related Content

What's hot (14)

Surds,
Surds,Surds,
Surds,
 
Ακολουθίες - Α.Π - Γ.Π 2021
Ακολουθίες - Α.Π - Γ.Π 2021Ακολουθίες - Α.Π - Γ.Π 2021
Ακολουθίες - Α.Π - Γ.Π 2021
 
Graph functions
Graph functionsGraph functions
Graph functions
 
Chapter 06
Chapter 06Chapter 06
Chapter 06
 
Smkts 2015 p1
Smkts 2015 p1Smkts 2015 p1
Smkts 2015 p1
 
Basic algebra for entrepreneurs
Basic algebra for entrepreneurs Basic algebra for entrepreneurs
Basic algebra for entrepreneurs
 
V2.0
V2.0V2.0
V2.0
 
4 chap
4 chap4 chap
4 chap
 
Diabolic Str8ts #4
Diabolic Str8ts #4Diabolic Str8ts #4
Diabolic Str8ts #4
 
Integrated 2 Section 3-1
Integrated 2 Section 3-1Integrated 2 Section 3-1
Integrated 2 Section 3-1
 
Str8ts Weekly Extreme #45 - Solution
Str8ts Weekly Extreme #45 - SolutionStr8ts Weekly Extreme #45 - Solution
Str8ts Weekly Extreme #45 - Solution
 
Add Maths 1
Add Maths 1Add Maths 1
Add Maths 1
 
09 trial jpwp_s2
09 trial jpwp_s209 trial jpwp_s2
09 trial jpwp_s2
 
19 Clt
19 Clt19 Clt
19 Clt
 

Viewers also liked (12)

Chapter 12
Chapter 12Chapter 12
Chapter 12
 
Chapter 09
Chapter 09Chapter 09
Chapter 09
 
Chapter 01
Chapter 01Chapter 01
Chapter 01
 
Chapter 04
Chapter 04Chapter 04
Chapter 04
 
Appendix a page_524
Appendix a page_524Appendix a page_524
Appendix a page_524
 
Chapter 14
Chapter 14Chapter 14
Chapter 14
 
Marketo web hook guide
Marketo web hook guideMarketo web hook guide
Marketo web hook guide
 
Chapter 15
Chapter 15Chapter 15
Chapter 15
 
Chapter 16
Chapter 16Chapter 16
Chapter 16
 
Chapter 07
Chapter 07Chapter 07
Chapter 07
 
Emirates Strategy and Implementation
Emirates Strategy and ImplementationEmirates Strategy and Implementation
Emirates Strategy and Implementation
 
プロジェクト進捗レポート
プロジェクト進捗レポートプロジェクト進捗レポート
プロジェクト進捗レポート
 

Similar to Chapter 10

Boas mathematical methods in the physical sciences 3ed instructors solutions...
Boas  mathematical methods in the physical sciences 3ed instructors solutions...Boas  mathematical methods in the physical sciences 3ed instructors solutions...
Boas mathematical methods in the physical sciences 3ed instructors solutions...
Praveen Prashant
 
Complete solutions-mathematical-methods-in-the-physical-sciences-3rd-edition
Complete solutions-mathematical-methods-in-the-physical-sciences-3rd-editionComplete solutions-mathematical-methods-in-the-physical-sciences-3rd-edition
Complete solutions-mathematical-methods-in-the-physical-sciences-3rd-edition
Aba Dula
 
Lesson 24: Area and Distances (worksheet solutions)
Lesson 24: Area and Distances (worksheet solutions)Lesson 24: Area and Distances (worksheet solutions)
Lesson 24: Area and Distances (worksheet solutions)
Matthew Leingang
 
5 indices & logarithms
5  indices & logarithms5  indices & logarithms
5 indices & logarithms
linusshy
 
S101-52國立新化高中(代理)
S101-52國立新化高中(代理)S101-52國立新化高中(代理)
S101-52國立新化高中(代理)
yustar1026
 
Hw4sol
Hw4solHw4sol
Hw4sol
uxxdqq
 

Similar to Chapter 10 (20)

Statistical controls for qc
Statistical controls for qcStatistical controls for qc
Statistical controls for qc
 
Statistical Tools for the Quality Control Laboratory and Validation Studies
Statistical Tools for the Quality Control Laboratory and Validation StudiesStatistical Tools for the Quality Control Laboratory and Validation Studies
Statistical Tools for the Quality Control Laboratory and Validation Studies
 
近似ベイズ計算によるベイズ推定
近似ベイズ計算によるベイズ推定近似ベイズ計算によるベイズ推定
近似ベイズ計算によるベイズ推定
 
Los atomos-t4
Los atomos-t4Los atomos-t4
Los atomos-t4
 
Antwoorden fourier and laplace transforms, manual solutions
Antwoorden   fourier and laplace transforms, manual solutionsAntwoorden   fourier and laplace transforms, manual solutions
Antwoorden fourier and laplace transforms, manual solutions
 
Boas mathematical methods in the physical sciences 3ed instructors solutions...
Boas  mathematical methods in the physical sciences 3ed instructors solutions...Boas  mathematical methods in the physical sciences 3ed instructors solutions...
Boas mathematical methods in the physical sciences 3ed instructors solutions...
 
Complete solutions-mathematical-methods-in-the-physical-sciences-3rd-edition
Complete solutions-mathematical-methods-in-the-physical-sciences-3rd-editionComplete solutions-mathematical-methods-in-the-physical-sciences-3rd-edition
Complete solutions-mathematical-methods-in-the-physical-sciences-3rd-edition
 
Ch15s
Ch15sCh15s
Ch15s
 
Nota math-spm
Nota math-spmNota math-spm
Nota math-spm
 
Lesson 24: Area and Distances (worksheet solutions)
Lesson 24: Area and Distances (worksheet solutions)Lesson 24: Area and Distances (worksheet solutions)
Lesson 24: Area and Distances (worksheet solutions)
 
Deber10
Deber10Deber10
Deber10
 
Math integration-homework help
Math integration-homework helpMath integration-homework help
Math integration-homework help
 
final19
final19final19
final19
 
Examen presencial 1
Examen presencial 1Examen presencial 1
Examen presencial 1
 
5 indices & logarithms
5  indices & logarithms5  indices & logarithms
5 indices & logarithms
 
Legendre
LegendreLegendre
Legendre
 
Sect1 1
Sect1 1Sect1 1
Sect1 1
 
S101-52國立新化高中(代理)
S101-52國立新化高中(代理)S101-52國立新化高中(代理)
S101-52國立新化高中(代理)
 
6 algebra
6 algebra6 algebra
6 algebra
 
Hw4sol
Hw4solHw4sol
Hw4sol
 

Recently uploaded

The basics of sentences session 3pptx.pptx
The basics of sentences session 3pptx.pptxThe basics of sentences session 3pptx.pptx
The basics of sentences session 3pptx.pptx
heathfieldcps1
 

Recently uploaded (20)

How to setup Pycharm environment for Odoo 17.pptx
How to setup Pycharm environment for Odoo 17.pptxHow to setup Pycharm environment for Odoo 17.pptx
How to setup Pycharm environment for Odoo 17.pptx
 
FSB Advising Checklist - Orientation 2024
FSB Advising Checklist - Orientation 2024FSB Advising Checklist - Orientation 2024
FSB Advising Checklist - Orientation 2024
 
How to Create and Manage Wizard in Odoo 17
How to Create and Manage Wizard in Odoo 17How to Create and Manage Wizard in Odoo 17
How to Create and Manage Wizard in Odoo 17
 
ICT role in 21st century education and it's challenges.
ICT role in 21st century education and it's challenges.ICT role in 21st century education and it's challenges.
ICT role in 21st century education and it's challenges.
 
General Principles of Intellectual Property: Concepts of Intellectual Proper...
General Principles of Intellectual Property: Concepts of Intellectual  Proper...General Principles of Intellectual Property: Concepts of Intellectual  Proper...
General Principles of Intellectual Property: Concepts of Intellectual Proper...
 
Food safety_Challenges food safety laboratories_.pdf
Food safety_Challenges food safety laboratories_.pdfFood safety_Challenges food safety laboratories_.pdf
Food safety_Challenges food safety laboratories_.pdf
 
Key note speaker Neum_Admir Softic_ENG.pdf
Key note speaker Neum_Admir Softic_ENG.pdfKey note speaker Neum_Admir Softic_ENG.pdf
Key note speaker Neum_Admir Softic_ENG.pdf
 
Holdier Curriculum Vitae (April 2024).pdf
Holdier Curriculum Vitae (April 2024).pdfHoldier Curriculum Vitae (April 2024).pdf
Holdier Curriculum Vitae (April 2024).pdf
 
UGC NET Paper 1 Mathematical Reasoning & Aptitude.pdf
UGC NET Paper 1 Mathematical Reasoning & Aptitude.pdfUGC NET Paper 1 Mathematical Reasoning & Aptitude.pdf
UGC NET Paper 1 Mathematical Reasoning & Aptitude.pdf
 
Sensory_Experience_and_Emotional_Resonance_in_Gabriel_Okaras_The_Piano_and_Th...
Sensory_Experience_and_Emotional_Resonance_in_Gabriel_Okaras_The_Piano_and_Th...Sensory_Experience_and_Emotional_Resonance_in_Gabriel_Okaras_The_Piano_and_Th...
Sensory_Experience_and_Emotional_Resonance_in_Gabriel_Okaras_The_Piano_and_Th...
 
On_Translating_a_Tamil_Poem_by_A_K_Ramanujan.pptx
On_Translating_a_Tamil_Poem_by_A_K_Ramanujan.pptxOn_Translating_a_Tamil_Poem_by_A_K_Ramanujan.pptx
On_Translating_a_Tamil_Poem_by_A_K_Ramanujan.pptx
 
REMIFENTANIL: An Ultra short acting opioid.pptx
REMIFENTANIL: An Ultra short acting opioid.pptxREMIFENTANIL: An Ultra short acting opioid.pptx
REMIFENTANIL: An Ultra short acting opioid.pptx
 
Kodo Millet PPT made by Ghanshyam bairwa college of Agriculture kumher bhara...
Kodo Millet  PPT made by Ghanshyam bairwa college of Agriculture kumher bhara...Kodo Millet  PPT made by Ghanshyam bairwa college of Agriculture kumher bhara...
Kodo Millet PPT made by Ghanshyam bairwa college of Agriculture kumher bhara...
 
Accessible Digital Futures project (20/03/2024)
Accessible Digital Futures project (20/03/2024)Accessible Digital Futures project (20/03/2024)
Accessible Digital Futures project (20/03/2024)
 
The basics of sentences session 3pptx.pptx
The basics of sentences session 3pptx.pptxThe basics of sentences session 3pptx.pptx
The basics of sentences session 3pptx.pptx
 
Fostering Friendships - Enhancing Social Bonds in the Classroom
Fostering Friendships - Enhancing Social Bonds  in the ClassroomFostering Friendships - Enhancing Social Bonds  in the Classroom
Fostering Friendships - Enhancing Social Bonds in the Classroom
 
ICT Role in 21st Century Education & its Challenges.pptx
ICT Role in 21st Century Education & its Challenges.pptxICT Role in 21st Century Education & its Challenges.pptx
ICT Role in 21st Century Education & its Challenges.pptx
 
HMCS Max Bernays Pre-Deployment Brief (May 2024).pptx
HMCS Max Bernays Pre-Deployment Brief (May 2024).pptxHMCS Max Bernays Pre-Deployment Brief (May 2024).pptx
HMCS Max Bernays Pre-Deployment Brief (May 2024).pptx
 
Python Notes for mca i year students osmania university.docx
Python Notes for mca i year students osmania university.docxPython Notes for mca i year students osmania university.docx
Python Notes for mca i year students osmania university.docx
 
Application orientated numerical on hev.ppt
Application orientated numerical on hev.pptApplication orientated numerical on hev.ppt
Application orientated numerical on hev.ppt
 

Chapter 10

  • 1. January 31, 2005 12:53 L24-ch10 Sheet number 1 Page number 424 black CHAPTER 10 Infinite Series EXERCISE SET 10.1 1 (−1)n−1 2n − 1 n2 1. (a) (b) (c) (d) 3n−1 3n−1 2n π 1/(n+1) 2. (a) (−r)n−1 ; (−r)n (b) (−1)n+1 rn ; (−1)n rn+1 3. (a) 2, 0, 2, 0 (b) 1, −1, 1, −1 (c) 2(1 + (−1)n ); 2 + 2 cos nπ 4. (a) (2n)! (b) (2n − 1)! n 5. 1/3, 2/4, 3/5, 4/6, 5/7, . . . ; lim = 1, converges n→+∞ n+2 n2 6. 1/3, 4/5, 9/7, 16/9, 25/11, . . . ; lim = +∞, diverges n→+∞ 2n + 1 7. 2, 2, 2, 2, 2, . . .; lim 2 = 2, converges n→+∞ 1 1 1 1 8. ln 1, ln , ln , ln , ln , . . . ; lim ln(1/n) = −∞, diverges 2 3 4 5 n→+∞ ln 1 ln 2 ln 3 ln 4 ln 5 9. , , , , ,...; 1 2 3 4 5 ln n 1 ln x lim = lim = 0 apply L’Hˆpital’s Rule to o , converges n→+∞ n n→+∞ n x 10. sin π, 2 sin(π/2), 3 sin(π/3), 4 sin(π/4), 5 sin(π/5), . . . ; sin(π/n) (−π/n2 ) cos(π/n) lim n sin(π/n) = lim = lim = π, converges n→+∞ n→+∞ 1/n n→+∞ −1/n2 11. 0, 2, 0, 2, 0, . . . ; diverges (−1)n+1 12. 1, −1/4, 1/9, −1/16, 1/25, . . . ; lim = 0, converges n→+∞ n2 13. −1, 16/9, −54/28, 128/65, −250/126, . . . ; diverges because odd-numbered terms approach −2, even-numbered terms approach 2. n 1 14. 1/2, 2/4, 3/8, 4/16, 5/32, . . . ; lim = lim n = 0, converges n→+∞ 2n n→+∞ 2 ln 2 1 15. 6/2, 12/8, 20/18, 30/32, 42/50, . . . ; lim (1 + 1/n)(1 + 2/n) = 1/2, converges n→+∞ 2 16. π/4, π 2 /42 , π 3 /43 , π 4 /44 , π 5 /45 , . . . ; lim (π/4)n = 0, converges n→+∞ 17. cos(3), cos(3/2), cos(1), cos(3/4), cos(3/5), . . . ; lim cos(3/n) = 1, converges n→+∞ 424
  • 2. January 31, 2005 12:53 L24-ch10 Sheet number 2 Page number 425 black Exercise Set 10.1 425 18. 0, −1, 0, 1, 0, . . . ; diverges x2 19. e−1 , 4e−2 , 9e−3 , 16e−4 , 25e−5 , . . . ; lim x2 e−x = lim = 0, so lim n2 e−n = 0, converges x→+∞ x→+∞ ex n→+∞ √ √ √ √ 20. 1, 10 − 2, 18 − 3, 28 − 4, 40 − 5, . . . ; 3n 3 3 lim ( n2 + 3n − n) = lim √ = lim = , converges n→+∞ n→+∞ n 2 + 3n + n n→+∞ 1 + 3/n + 1 2 x x+3 21. 2, (5/3)2 , (6/4)3 , (7/5)4 , (8/6)5 , . . . ; let y = , converges because x+1 x+3 ln 2x2 n+3 n lim ln y = lim x + 1 = lim = 2, so lim = e2 x→+∞ x→+∞ 1/x x→+∞ (x + 1)(x + 3) n→+∞ n + 1 22. −1, 0, (1/3)3 , (2/4)4 , (3/5)5 , . . . ; let y = (1 − 2/x)x , converges because ln(1 − 2/x) −2 lim ln y = lim = lim = −2, lim (1 − 2/n)n = lim y = e−2 x→+∞ x→+∞ 1/x x→+∞ 1 − 2/x n→+∞ x→+∞ +∞ 2n − 1 2n − 1 23. ; lim = 1, converges 2n n=1 n→+∞ 2n +∞ n−1 n−1 24. ; lim = 0, converges n2 n=1 n→+∞ n2 +∞ 1 (−1)n−1 25. (−1)n−1 ; lim = 0, converges 3n n=1 n→+∞ 3n +∞ 26. {(−1)n n}n=1 ; diverges because odd-numbered terms tend toward −∞, even-numbered terms tend toward +∞. +∞ 1 1 1 1 27. − ; lim − = 0, converges n n+1 n=1 n→+∞ n n+1 +∞ 28. 3/2n−1 ; lim 3/2n−1 n=1 n→+∞ = 0, converges √ √ +∞ 29. n + 1 − n + 2 n=1 ; converges because √ √ (n + 1) − (n + 2) −1 lim ( n + 1 − n + 2) = lim √ √ = lim √ √ =0 n→+∞ n→+∞ n+1+ n+2 n→+∞ n+1+ n+2 +∞ 30. (−1)n+1 /3n+4 ; lim (−1)n+1 /3n+4 n=1 n→+∞ = 0, converges +1 k even 31. an = oscillates; there is no limit point which attracts all of the an . −1 k odd bn = cos n; the terms lie all over the interval [−1, 1] without any limit. 32. (a) No, because given N > 0, all values of f (x) are greater than N provided x is close enough to zero. But certainly the terms 1/n will be arbitrarily close to zero, and when so then f (1/n) > N , so f (1/n) cannot converge. (b) f (x) = sin(π/x). Then f = 0 when x = 1/n and f = 0 otherwise; indeed, the values of f are located all over the interval [−1, 1].
  • 3. January 31, 2005 12:53 L24-ch10 Sheet number 3 Page number 426 black 426 Chapter 10 n, n odd 1/n, n odd 33. (a) 1, 2, 1, 4, 1, 6 (b) an = (c) an = n 1/2 , n even 1/(n + 1), n even (d) In Part (a) the sequence diverges, since the even terms diverge to +∞ and the odd terms equal 1; in Part (b) the sequence diverges, since the odd terms diverge to +∞ and the even terms tend to zero; in Part (c) lim an = 0. n→+∞ 34. The even terms are zero, so the odd terms must converge to zero, and this is true if and only if lim bn = 0, or −1 < b < 1. n→+∞ √ √ n 35. lim n n = 1, so lim n3 = 13 = 1 n→+∞ n→+∞ 1 a 1 a √ √ 37. lim xn+1 = lim xn + or L = L+ , 2L2 − L2 − a = 0, L = a (we reject − a n→+∞ 2 n→+∞ xn 2 L because xn > 0, thus L ≥ 0.) √ 38. (a) an+1 = 6 + an √ √ (b) lim an+1 = lim 6 + an , L = 6 + L, L2 − L − 6 = 0, (L − 3)(L + 2) = 0, n→+∞ n→+∞ L = −2 (reject, because the terms in the sequence are positive) or L = 3; lim an = 3. n→+∞ 1 2 1 2 3 1 2 3 4 3 2 5 39. (a) 1, + , + + , + + + = 1, , , 4 4 9 9 9 16 16 16 16 4 3 8 1 1 1 1n+1 (c) an = (1 + 2 + · · · + n) = 2 n(n + 1) = , lim an = 1/2 n2 n 2 2 n n→+∞ 1 4 1 4 9 1 4 9 16 5 14 15 40. (a) 1, + , + + , + + + = 1, , , 8 8 27 27 27 64 64 64 64 8 27 32 1 2 1 1 1 (n + 1)(2n + 1) (c) an = (1 + 22 + · · · + n2 ) = 3 n(n + 1)(2n + 1) = , n3 n 6 6 n2 1 lim an = lim (1 + 1/n)(2 + 1/n) = 1/3 n→+∞ n→+∞ 6 sin2 n 1 41. Let an = 0, bn = , cn = ; then an ≤ bn ≤ cn , lim an = lim cn = 0, so lim bn = 0. n n n→+∞ n→+∞ n→+∞ n n n 1+n 3 n/2 + n 42. Let an = 0, bn = , cn = ; then (for n ≥ 2), an ≤ bn ≤ = cn , 2n 4 2n lim an = lim cn = 0, so lim bn = 0. n→+∞ n→+∞ n→+∞ n 43. (a) a1 = (0.5)2 , a2 = a2 = (0.5)4 , . . . , an = (0.5)2 1 n (c) lim an = lim e2 ln(0.5) = 0, since ln(0.5) < 0. n→+∞ n→+∞ (d) Replace 0.5 in Part (a) with a0 ; then the sequence converges for −1 ≤ a0 ≤ 1, because if a0 = ±1, then an = 1 for n ≥ 1; if a0 = 0 then an = 0 for n ≥ 1; and if 0 < |a0 | < 1 then n−1 a1 = a2 > 0 and lim an = lim e2 0 ln a1 = 0 since 0 < a1 < 1. This same argument n→+∞ n→+∞ proves divergence to +∞ for |a| > 1 since then ln a1 > 0.
  • 4. January 31, 2005 12:53 L24-ch10 Sheet number 4 Page number 427 black Exercise Set 10.1 427 44. f (0.2) = 0.4, f (0.4) = 0.8, f (0.8) = 0.6, f (0.6) = 0.2 and then the cycle repeats, so the sequence does not converge. 45. (a) 30 0 5 0 ln(2x + 3x ) 2x ln 2 + 3x ln 3 (b) Let y = (2x + 3x )1/x , lim ln y = lim = lim x→+∞ x→+∞ x x→+∞ 2x + 3x x (2/3) ln 2 + ln 3 = lim = ln 3, so lim (2n + 3n )1/n = eln 3 = 3 x→+∞ (2/3)x + 1 n→+∞ Alternate proof: 3 = (3n )1/n < (2n +3n )1/n < (2·3n )1/n = 3·21/n . Then apply the Squeezing Theorem. 46. Let f (x) = 1/(1 + x), 0 ≤ x ≤ 1. Take ∆xk = 1/n and x∗ = k/n then k n n 1 1 1 1 1 an = (1/n) = ∆xk so lim an = dx = ln(1 + x) = ln 2 1 + (k/n) 1 + x∗ k n→+∞ 0 1+x 0 k=1 k=1 n 1 1 ln n ln n 1 47. an = dx = , lim an = lim = lim = 0, n−1 1 x n − 1 n→+∞ n→+∞ n − 1 n→+∞ n ln n apply L’Hˆpital’s Rule to o , converges n−1 an+2 an 48. (a) If n ≥ 1, then an+2 = an+1 + an , so =1+ . an+1 an+1 (c) With L = lim (an+2 /an+1 ) = lim (an+1 /an ), L = 1 + 1/L, L2 − L − 1 = 0, n→+∞ n→+∞ √ √ L = (1 ± 5)/2, so L = (1 + 5)/2 because the limit cannot be negative. 1 1 49. −0 = < if n > 1/ n n (a) 1/ = 1/0.5 = 2, N = 3 (b) 1/ = 1/0.1 = 10, N = 11 (c) 1/ = 1/0.001 = 1000, N = 1001 n 1 50. −1 = < if n + 1 > 1/ , n > 1/ − 1 n+1 n+1 (a) 1/ − 1 = 1/0.25 − 1 = 3, N = 4 (b) 1/ − 1 = 1/0.1 − 1 = 9, N = 10 (c) 1/ − 1 = 1/0.001 − 1 = 999, N = 1000 1 1 51. (a) −0 = < if n > 1/ , choose any N > 1/ . n n n 1 (b) −1 = < if n > 1/ − 1, choose any N > 1/ − 1. n+1 n+1
  • 5. January 31, 2005 12:53 L24-ch10 Sheet number 5 Page number 428 black 428 Chapter 10 52. If |r| < 1 then lim rn = 0; if r > 1 then lim rn = +∞, if r < −1 then rn oscillates between n→+∞ n→+∞ positive and negative values that grow in magnitude so lim rn does not exist for |r| > 1; if r = 1 n→+∞ then lim 1n = 1; if r = −1 then (−1)n oscillates between −1 and 1 so lim (−1)n does not exist. n→+∞ n→+∞ EXERCISE SET 10.2 1 1 1 1. an+1 − an = − =− < 0 for n ≥ 1, so strictly decreasing. n+1 n n(n + 1) 1 1 1 2. an+1 − an = (1 − ) − (1 − ) = > 0 for n ≥ 1, so strictly increasing. n+1 n n(n + 1) n+1 n 1 3. an+1 − an = − = > 0 for n ≥ 1, so strictly increasing. 2n + 3 2n + 1 (2n + 1)(2n + 3) n+1 n 1 4. an+1 − an = − =− < 0 for n ≥ 1, so strictly decreasing. 4n + 3 4n − 1 (4n − 1)(4n + 3) 5. an+1 − an = (n + 1 − 2n+1 ) − (n − 2n ) = 1 − 2n < 0 for n ≥ 1, so strictly decreasing. 6. an+1 − an = [(n + 1) − (n + 1)2 ] − (n − n2 ) = −2n < 0 for n ≥ 1, so strictly decreasing. an+1 (n + 1)/(2n + 3) (n + 1)(2n + 1) 2n2 + 3n + 1 7. = = = > 1 for n ≥ 1, so strictly increasing. an n/(2n + 1) n(2n + 3) 2n2 + 3n an+1 2n+1 1 + 2n 2 + 2n+1 1 8. = · = =1+ > 1 for n ≥ 1, so strictly increasing. an 1 + 2n+1 2n 1 + 2n+1 1 + 2n+1 an+1 (n + 1)e−(n+1) 9. = = (1 + 1/n)e−1 < 1 for n ≥ 1, so strictly decreasing. an ne−n an+1 10n+1 (2n)! 10 10. = · = < 1 for n ≥ 1, so strictly decreasing. an (2n + 2)! 10n (2n + 2)(2n + 1) an+1 (n + 1)n+1 n! (n + 1)n 11. = · n = = (1 + 1/n)n > 1 for n ≥ 1, so strictly increasing. an (n + 1)! n nn 2 an+1 5n+1 2n 5 12. = (n+1)2 · n = 2n+1 < 1 for n ≥ 1, so strictly decreasing. an 2 5 2 13. f (x) = x/(2x + 1), f (x) = 1/(2x + 1)2 > 0 for x ≥ 1, so strictly increasing. 14. f (x) = 3 − 1/x, f (x) = 1/x2 > 0 for x ≥ 1, so strictly increasing. 1 + 1/x 15. f (x) = 1/(x + ln x), f (x) = − < 0 for x ≥ 1, so strictly decreasing. (x + ln x)2 16. f (x) = xe−2x , f (x) = (1 − 2x)e−2x < 0 for x ≥ 1, so strictly decreasing. ln(x + 2) 1 − ln(x + 2) 17. f (x) = , f (x) = < 0 for x ≥ 1, so strictly decreasing. x+2 (x + 2)2 18. f (x) = tan−1 x, f (x) = 1/(1 + x2 ) > 0 for x ≥ 1, so strictly increasing.
  • 6. January 31, 2005 12:53 L24-ch10 Sheet number 6 Page number 429 black Exercise Set 10.2 429 19. f (x) = 2x2 − 7x, f (x) = 4x − 7 > 0 for x ≥ 2, so eventually strictly increasing. 20. f (x) = x3 − 4x2 , f (x) = 3x2 − 8x = x(3x − 8) > 0 for x ≥ 3, so eventually strictly increasing. x 10 − x2 21. f (x) = , f (x) = 2 < 0 for x ≥ 4, so eventually strictly decreasing. x2 + 10 (x + 10)2 17 x2 − 17 22. f (x) = x + , f (x) = > 0 for x ≥ 5, so eventually strictly increasing. x x2 an+1 (n + 1)! 3n n+1 23. = · = > 1 for n ≥ 3, so eventually strictly increasing. an 3n+1 n! 3 24. f (x) = x5 e−x , f (x) = x4 (5 − x)e−x < 0 for x ≥ 6, so eventually strictly decreasing. 25. (a) Yes: a monotone sequence is increasing or decreasing; if it is increasing, then it is increas- ing and bounded above, so by Theorem 10.2.3 it converges; if decreasing, then use Theo- rem 10.2.4. The limit lies in the interval [1, 2]. (b) Such a sequence may converge, in which case, by the argument in Part (a), its limit is ≤ 2. But convergence may not happen: for example, the sequence {−n}+∞ diverges. n=1 26. The sequence {1} is monotone but not eventually strictly monotone. Let {xn }+∞ be a sequence n=1 that is monotone (let’s say increasing) but not eventually strictly monotone. Then there exists some N0 > 0 such that for all n ≥ N0 we have xn ≤ xn+1 . Yet for any N > 0 it is not true that if n ≥ N then xn < xn+1 . Since the sequence is monotone increasing we must have xn ≤ xn+1 for all n, yet infinitely many times it is false that xn < xn+1 . We conclude that for some n > N, xn = xn+1 . Rephrasing, we can say that for any n > 0, there exists k, say kn , such that xkn = xkn +1 . The sequence has infinitely many pairs of repeated terms. |x|n+1 |x| |x|n |x| 27. (a) an+1 = = = an (n + 1)! n + 1 n! n+1 (b) an+1 /an = |x|/(n + 1) < 1 if n > |x| − 1. (c) From Part (b) the sequence is eventually decreasing, and it is bounded below by 0, so by Theorem 10.2.4 it converges. |x| (d) If lim an = L then from Part (a), L = L = 0. n→+∞ lim (n + 1) n→+∞ |x|n (e) lim = lim an = 0 n→+∞ n! n→+∞ √ √ √ 28. (a) 2, 2 + 2, 2 + 2 + 2 √ √ √ √ √ (b) a1 = 2 < 2 so a2 = 2 + a1 < 2 + 2 = 2, a3 = 2 + a2 < 2 + 2 = 2, and so on indefinitely. (c) a2 − a2 = (2 + an ) − a2 = 2 + an − a2 = (2 − an )(1 + an ) n+1 n n n (d) an > 0 and, from Part (b), an < 2 so 2 − an > 0 and 1 + an > 0 thus, from Part (c), a2 − a2 > 0, an+1 − an > 0, an+1 > an ; {an } is a strictly increasing sequence. n+1 n (e) The sequence is increasing and has 2 as an upper bound so it must converge to a limit L, √ √ lim an+1 = lim 2 + an , L = 2 + L, L2 − L − 2 = 0, (L − 2)(L + 1) = 0 n→+∞ n→+∞ thus lim an = 2. n→+∞
  • 7. January 31, 2005 12:53 L24-ch10 Sheet number 7 Page number 430 black 430 Chapter 10 √ 29. (a) If f (x) = 1 (x + 3/x), then f (x) = (x2 − 3)/(2x2 ) and f (x) = 0 for x = 3; the minimum 2 √ √ √ √ value of f (x) for x > 0 is f ( 3) = 3. Thus f (x) ≥ 3 for x > 0 and hence an ≥ 3 for n ≥ 2. √ (b) an+1 − an = (3 − a2 )/(2an ) ≤ 0 for n ≥ 2 since an ≥ 3 for n ≥ 2; {an } is eventually n decreasing. √ (c) 3 is a lower bound for an so {an } converges; lim an+1 = lim 1 (an + 3/an ), n→+∞ 2 √ n→+∞ L = 2 (L + 3/L), L − 3 = 0, L = 3. 1 2 30. (a) The altitudes of the rectangles are ln k for k = 2 to n, and their bases all have length 1 so the sum of their areas is ln 2 + ln 3 + · · · + ln n = ln(2 · 3 · · · n) = ln n!. The area under the n n+1 curve y = ln x for x in the interval [1, n] is ln x dx, and ln x dx is the area for x in 1 1 n n+1 the interval [1, n + 1] so, from the figure, ln x dx < ln n! < ln x dx. 1 1 n n n+1 (b) ln x dx = (x ln x − x) = n ln n − n + 1 and ln x dx = (n + 1) ln(n + 1) − n so from 1 1 1 Part (a), n ln n − n + 1 < ln n! < (n + 1) ln(n + 1) − n, en ln n−n+1 < n! < e(n+1) ln(n+1)−n , nn (n + 1)n+1 en ln n e1−n < n! < e(n+1) ln(n+1) e−n , n−1 < n! < e en nn 1/n √ (n + 1)n+1 n 1/n (c) From Part (b), < n! < , en−1 en √ n √ n (n + 1)1+1/n 1 n n! (1 + 1/n)(n + 1)1/n < n! < , 1−1/n < < , e1−1/n e e n e √ 1 1 (1 + 1/n)(n + 1)1/n 1 n n! 1 but → and → as n → +∞ (why?), so lim = . e1−1/n e e e n→+∞ n e nn √ n n n √ n 31. n! > n−1 , n! > 1−1/n , lim 1−1/n = +∞ so lim n! = +∞. e e n→+∞ e n→+∞ EXERCISE SET 10.3 62 312 2 − 2(1/5)n 5 5 1. (a) s1 = 2, s2 = 12/5, s3 = , s4 = sn = = − (1/5)n , 25 125 1 − 1/5 2 2 5 lim sn = , converges n→+∞ 2 1 3 7 15 (1/4) − (1/4)2n 1 1 (b) s1 = , s2 = , s3 = , s4 = sn = = − + (2n ), 4 4 4 4 1−2 4 4 lim sn = +∞, diverges n→+∞ 1 1 1 1 1 3 1 (c) = − , s1 = , s2 = , s3 = , s4 = ; (k + 1)(k + 2) k+1 k+2 6 4 10 3 1 1 1 sn = − , lim sn = , converges 2 n + 2 n→+∞ 2
  • 8. January 31, 2005 12:53 L24-ch10 Sheet number 8 Page number 431 black Exercise Set 10.3 431 2. (a) s1 = 1/4, s2 = 5/16, s3 = 21/64, s4 = 85/256 n−1 n 1 1 1 1 1 − (1/4)n 1 1 1 sn = 1+ + ··· + = = 1− ; lim sn = 4 4 4 4 1 − 1/4 3 4 n→+∞ 3 4n − 1 (b) s1 = 1, s2 = 5, s3 = 21, s4 = 85; sn = , diverges 3 (c) s1 = 1/20, s2 = 1/12, s3 = 3/28, s4 = 1/8; n 1 1 1 1 sn = − = − , lim sn = 1/4 k+3 k+4 4 n + 4 n→+∞ k=1 1 3. geometric, a = 1, r = −3/4, sum = = 4/7 1 − (−3/4) (2/3)3 4. geometric, a = (2/3)3 , r = 2/3, sum = = 8/9 1 − 2/3 7 5. geometric, a = 7, r = −1/6, sum = =6 1 + 1/6 6. geometric, r = −3/2, diverges n 1 1 1 1 7. sn = − = − , lim sn = 1/3 k+2 k+3 3 n + 3 n→+∞ k=1 n 1 1 1 1 8. sn = − k+1 = − , lim sn = 1/2 2k 2 2 2n+1 n→+∞ k=1 n 1/3 1/3 1 1/3 9. sn = − = − , lim sn = 1/6 3k − 1 3k + 2 6 3n + 2 n→+∞ k=1 n+1 n+1 n+1 1/2 1/2 1 1 1 10. sn = − = − k−1 k+1 2 k−1 k+1 k=2 k=2 k=2 n+1 n+3 1 1 1 1 1 1 1 3 = − = 1+ − − ; lim sn = 2 k−1 k−1 2 2 n+1 n+2 n→+∞ 4 k=2 k=4 ∞ ∞ 1 11. = 1/k, the harmonic series, so the series diverges. k−2 k=3 k=1 (e/π)4 e4 12. geometric, a = (e/π)4 , r = e/π < 1, sum = = 3 1 − e/π π (π − e) ∞ ∞ k−1 4k+2 4 64 13. = 64 ; geometric, a = 64, r = 4/7, sum = = 448/3 7k−1 7 1 − 4/7 k=1 k=1 14. geometric, a = 125, r = 125/7, diverges 15. (a) Exercise 5 (b) Exercise 3 (c) Exercise 7 (d) Exercise 9
  • 9. January 31, 2005 12:53 L24-ch10 Sheet number 9 Page number 432 black 432 Chapter 10 16. (a) Exercise 10 (b) Exercise 6 (c) Exercise 4 (d) Exercise 8 0.4 17. 0.4444 · · · = 0.4 + 0.04 + 0.004 + · · · = = 4/9 1 − 0.1 0.9 18. 0.9999 · · · = 0.9 + 0.09 + 0.009 + · · · = =1 1 − 0.1 0.37 19. 5.373737 · · · = 5 + 0.37 + 0.0037 + 0.000037 + · · · = 5 + = 5 + 37/99 = 532/99 1 − 0.01 0.00014 44663 20. 0.451141414 · · · = 0.451 + 0.00014 + 0.0000014 + 0.000000014 + · · · = 0.451 + = 1 − 0.01 99000 21. 0.a1 a2 · · · an 9999 · · · = 0.a1 a2 · · · an + 0.9 (10−n ) + 0.09 (10−n ) + · · · 0.9 (10−n ) = 0.a1 a2 · · · an + = 0.a1 a2 · · · an + 10−n 1 − 0.1 = 0.a1 a2 · · · (an + 1) = 0.a1 a2 · · · (an + 1) 0000 · · · 22. The series converges to 1/(1 − x) only if −1 < x < 1. 3 3 3 3 3 3 23. d = 10 + 2 · · 10 + 2 · · · 10 + 2 · · · · 10 + · · · 4 4 4 4 4 4 2 3 3 3 3 20(3/4) = 10 + 20 + 20 + 20 + · · · = 10 + = 10 + 60 = 70 meters 4 4 4 1 − 3/4 3 3 3 2 n 1 1 1 1 1 1 24. volume = 13 + + + ··· + + ··· = 1 + + + ··· + + ··· 2 4 2n 8 8 8 1 = = 8/7 1 − (1/8) 1 2 3 n 1 2 3 n 1 25. (a) sn = ln + ln + ln + · · · + ln = ln · · ··· = ln = − ln(n + 1), 2 3 4 n+1 2 3 4 n+1 n+1 lim sn = −∞, series diverges. n→+∞ k2 − 1 (k − 1)(k + 1) k−1 k+1 k−1 k (b) ln(1 − 1/k 2 ) = ln = ln = ln + ln = ln − ln , k2 k2 k k k k+1 n+1 k−1 k sn = ln − ln k k+1 k=2 1 2 2 3 3 4 n n+1 = ln − ln + ln − ln + ln − ln + · · · + ln − ln 2 3 3 4 4 5 n+1 n+2 1 n+1 1 = ln − ln , lim sn = ln = − ln 2 2 n + 2 n→+∞ 2 ∞ 1 1 26. (a) (−1)k xk = 1 − x + x2 − x3 + · · · = = if | − x| < 1, |x| < 1, −1 < x < 1. 1 − (−x) 1+x k=0 ∞ 1 1 (b) (x − 3)k = 1 + (x − 3) + (x − 3)2 + · · · = = if |x − 3| < 1, 2 < x < 4. 1 − (x − 3) 4−x k=0 ∞ 1 1 (c) (−1)k x2k = 1−x2 +x4 −x6 +· · · = = if |−x2 | < 1, |x| < 1, −1 < x < 1. 1 − (−x2 ) 1 + x2 k=0
  • 10. January 31, 2005 12:53 L24-ch10 Sheet number 10 Page number 433 black Exercise Set 10.3 433 27. (a) Geometric series, a = x, r = −x2 . Converges for | − x2 | < 1, |x| < 1; x x S= = . 1 − (−x 2) 1 + x2 (b) Geometric series, a = 1/x2 , r = 2/x. Converges for |2/x| < 1, |x| > 2; 1/x2 1 S= = 2 . 1 − 2/x x − 2x (c) Geometric series, a = e−x , r = e−x . Converges for |e−x | < 1, e−x < 1, ex > 1, x > 0; e−x 1 S= −x = x . 1−e e −1 1 1 28. Geometric series, a = sin x, r = − sin x. Converges for | − sin x| < 1, | sin x| < 2, 2 2 sin x 2 sin x so converges for all values of x. S = = . 1 2 + sin x 1 + sin x 2 1 1 1 1 1 1 1 1 1 1 1 1 1 29. a2 = a1 + , a3 = a2 + = 2 a1 + 2 + , a4 = a3 + = 3 a1 + 3 + 2 + , 2 2 2 2 2 2 2 2 2 2 2 2 2 1 1 1 1 1 1 1 1 1 1 1 a5 = a4 + = 4 a1 + 4 + 3 + 2 + , . . . , an = n−1 a1 + n−1 + n−2 + · · · + , 2 2 2 2 2 2 2 2 2 2 2 ∞ n a1 1 1/2 lim an = lim + =0+ =1 n→+∞ n→+∞ 2n−1 n=1 2 1 − 1/2 √ √ √ √ k+1− k k+1− k 1 1 30. √ = √ √ =√ −√ , k 2+k k k+1 k k+1 n 1 1 1 1 1 1 1 1 sn = √ −√ = √ −√ + √ −√ + √ −√ k=1 k k+1 1 2 2 3 3 4 1 1 1 +··· + √ −√ =1− √ ; lim sn = 1 n n+1 n + 1 n→+∞ 31. sn = (1 − 1/3) + (1/2 − 1/4) + (1/3 − 1/5) + (1/4 − 1/6) + · · · + [1/n − 1/(n + 2)] = (1 + 1/2 + 1/3 + · · · + 1/n) − (1/3 + 1/4 + 1/5 + · · · + 1/(n + 2)) = 3/2 − 1/(n + 1) − 1/(n + 2), lim sn = 3/2 n→+∞ n n n n 1 1/2 1/2 1 1 1 32. sn = = − = − k(k + 2) k k+2 2 k k+2 k=1 k=1 k=1 k=1 n n+2 1 1 1 1 1 1 1 3 = − = 1+ − − ; lim sn = 2 k k 2 2 n+1 n+2 n→+∞ 4 k=1 k=3 n n n n 1 1/2 1/2 1 1 1 33. sn = = − = − (2k − 1)(2k + 1) 2k − 1 2k + 1 2 2k − 1 2k + 1 k=1 k=1 k=1 k=1 n n+1 1 1 1 1 1 1 = − = 1− ; lim sn = 2 2k − 1 2k − 1 2 2n + 1 n→+∞ 2 k=1 k=2
  • 11. January 31, 2005 12:53 L24-ch10 Sheet number 11 Page number 434 black 434 Chapter 10 34. P0 P1 = a sin θ, a sin u cos u P1 P1 P2 = a sin θ cos θ, P2 P3 = a sin θ cos2 θ, P3 a sin u cos2 u a sin u P3 P4 = a sin θ cos3 θ, . . . u a sin u cos3 u u (see figure) Each sum is a geometric series. u P P0 P2 P4 a a sin θ (a) P0 P1 + P1 P2 + P2 P3 + · · · = a sin θ + a sin θ cos θ + a sin θ cos2 θ + · · · = 1 − cos θ (b) P0 P1 + P2 P3 + P4 P5 + · · · = a sin θ + a sin θ cos2 θ + a sin θ cos4 θ + · · · a sin θ a sin θ = = = a csc θ 1 − cos2 θ sin2 θ (c) P1 P2 + P3 P4 + P5 P6 + · · · = a sin θ cos θ + a sin θ cos3 θ + · · · a sin θ cos θ a sin θ cos θ = = = a cot θ 1 − cos2 θ sin2 θ θ θ θ θ θ/2 35. By inspection, − + − + ··· = = θ/3 2 4 8 16 1 − (−1/2) 1 36. A1 + A2 + A3 + · · · = 1 + 1/2 + 1/4 + · · · = =2 1 − (1/2) 2k A 2k B 2k 3k+1 − 2k+1 A + 2k 3k − 2k B 37. (b) + k+1 = 3k − 2 k 3 − 2k+1 (3k − 2k ) (3k+1 − 2k+1 ) 3 · 6k − 2 · 22k A + 6k − 22k B (3A + B)6k − (2A + B)22k = = (3k − 2k ) (3k+1 − 2k+1 ) (3k − 2k ) (3k+1 − 2k+1 ) so 3A + B = 1 and 2A + B = 0, A = 1 and B = −2. n n 2k 2k+1 2k (c) sn = − k+1 = (ak − ak+1 ) where ak = . 3k − 2 k 3 − 2k+1 3k − 2 k k=1 k=1 But sn = (a1 − a2 ) + (a2 − a3 ) + (a3 − a4 ) + · · · + (an − an+1 ) which is a telescoping sum, 2n+1 (2/3)n+1 sn = a1 − an+1 = 2 − , lim sn = lim 2 − = 2. 3n+1 − 2n+1 n→+∞ n→+∞ 1 − (2/3)n+1 ∞ 1 1 1 38. (a) geometric; 18/5 (b) geometric; diverges (c) − = 1/2 2 2k − 1 2k + 1 k=1 EXERCISE SET 10.4 ∞ ∞ ∞ 1 1/2 1 1/4 1 1 1. (a) = = 1; = = 1/3; + k = 1 + 1/3 = 4/3 2k 1 − 1/2 4k 1 − 1/4 2k 4 k=1 k=1 k=1 ∞ ∞ 1 1/5 1 (b) = = 1/4; =1 (Example 5, Section 10 .4); 5 k 1 − 1/5 k(k + 1) k=1 k=1 ∞ 1 1 − = 1/4 − 1 = −3/4 5k k(k + 1) k=1
  • 12. January 31, 2005 12:53 L24-ch10 Sheet number 12 Page number 435 black Exercise Set 10.4 435 ∞ ∞ 1 7 7/10 2. (a) = 3/4 (Exercise 10, Section 10.4); = = 7/9; k2 − 1 10k−1 1 − 1/10 k=2 k=2 ∞ 1 7 so − = 3/4 − 7/9 = −1/36 k 2 − 1 10k−1 k=2 ∞ 9/7 (b) with a = 9/7, r = 3/7, geometric, 7−k 3k+1 = = 9/4; 1 − (3/7) k=1 ∞ 2k+1 4/5 with a = 4/5, r = 2/5, geometric, = = 4/3; 5k 1 − (2/5) k=1 ∞ k+1 2 7−k 3k+1 − = 9/4 − 4/3 = 11/12 5k k=1 3. (a) p = 3, converges (b) p = 1/2, diverges (c) p = 1, diverges (d) p = 2/3, diverges 4. (a) p = 4/3, converges (b) p = 1/4, diverges (c) p = 5/3, converges (d) p = π, converges k k2 + k + 3 1 1 5. (a) lim 2+1 = ; the series diverges. (b) lim 1+ = e; the series diverges. k→+∞ 2k 2 k→+∞ k 1 (c) lim cos kπ does not exist; (d) lim = 0; no information k→+∞ k→+∞ k! the series diverges. k 6. (a) lim = 0; no information (b) lim ln k = +∞; the series diverges. k→+∞ ek k→+∞ √ 1 k (c) lim √ = 0; no information (d) lim √ = 1; the series diverges. k→+∞ k k→+∞ k+3 +∞ 1 1 7. (a) = lim ln(5x + 2) = +∞, the series diverges by the Integral Test. 1 5x + 2 →+∞ 5 1 +∞ 1 1 1 (b) 2 dx = lim tan−1 3x = π/2 − tan−1 3 , 1 1 + 9x →+∞ 3 1 3 the series converges by the Integral Test. +∞ x 1 8. (a) 2 dx = lim ln(1 + x2 ) = +∞, the series diverges by the Integral Test. 1 1+x →+∞ 2 1 +∞ √ √ (b) (4 + 2x)−3/2 dx = lim −1/ 4 + 2x = 1/ 6, 1 →+∞ 1 the series converges by the Integral Test. ∞ ∞ 1 1 9. = , diverges because the harmonic series diverges. k+6 k k=1 k=7 ∞ ∞ 3 3 1 10. = , diverges because the harmonic series diverges. 5k 5 k k=1 k=1 ∞ ∞ 1 1 11. √ = √ , diverges because the p-series with p = 1/2 ≤ 1 diverges. k=1 k + 5 k=6 k
  • 13. January 31, 2005 12:53 L24-ch10 Sheet number 13 Page number 436 black 436 Chapter 10 1 12. lim = 1, the series diverges because lim uk = 1 = 0. k→+∞ e1/k k→+∞ +∞ 3 13. (2x − 1)−1/3 dx = lim (2x − 1)2/3 = +∞, the series diverges by the Integral Test. 1 →+∞ 4 1 +∞ ln x ln x 1 14. is decreasing for x ≥ e, and = lim (ln x)2 = +∞, x 3 x →+∞ 2 3 so the series diverges by the Integral Test. k 1 15. lim = lim = +∞, the series diverges because lim uk = 0. k→+∞ ln(k + 1) k→+∞ 1/(k + 1) k→+∞ +∞ 1 xe−x dx = lim − e−x = e−1 /2, the series converges by the Integral Test. 2 2 16. 1 →+∞ 2 1 17. lim (1 + 1/k)−k = 1/e = 0, the series diverges. k→+∞ k2 + 1 18. lim = 1 = 0, the series diverges. k→+∞ k 2 + 3 +∞ tan−1 x 1 2 19. 2 dx = lim tan−1 x = 3π 2 /32, the series converges by the Integral Test, since 1 1+x →+∞ 2 1 d tan−1 x 1 − 2x tan−1 x = < 0 for x ≥ 1. dx 1 + x2 (1 + x2 )2 +∞ 1 20. √ dx = lim sinh−1 x = +∞, the series diverges by the Integral Test. 1 x2 +1 →+∞ 1 21. lim k 2 sin2 (1/k) = 1 = 0, the series diverges. k→+∞ +∞ 1 x2 e−x dx = lim − e−x = e−1 /3, 3 3 22. 1 →+∞ 3 1 the series converges by the Integral Test (x2 e−x is decreasing for x ≥ 1). 3 ∞ 23. 7 k −1.01 , p-series with p > 1, converges k=5 +∞ 24. sech2 x dx = lim tanh x = 1 − tanh(1), the series converges by the Integral Test. 1 →+∞ 1 +∞ 1 dx 25. p is decreasing for x ≥ ep , so use the Integral Test with to get x(ln x) ep x(ln x)p  1−p  +∞  if p < 1 (ln x) lim ln(ln x) = +∞ if p = 1, lim = 1−p →+∞ ep →+∞ 1 − p ep  p  if p > 1 p−1 Thus the series converges for p > 1.
  • 14. January 31, 2005 12:53 L24-ch10 Sheet number 14 Page number 437 black Exercise Set 10.4 437 26. If p > 0 set g(x) = x(ln x)[ln(ln x)]p , g (x) = (ln(ln x))p−1 [(1 + ln x) ln(ln x) + p], and, for x > ee , +∞ dx g (x) > 0, thus 1/g(x) is decreasing for x > ee ; use the Integral Test with ee x(ln x)[ln(ln x)]p to get  1−p  +∞  if p < 1, [ln(ln x)] lim ln[ln(ln x)] = +∞ if p = 1, lim = →+∞ ee →+∞ 1−p ee  1  if p > 1 p−1 [ln(ln x)]p 1 Thus the series converges for p > 1 and diverges for 0 < p ≤ 1. If p ≤ 0 then ≥ x ln x x ln x for x > ee so the series diverges. 27. Suppose Σ(uk + vk ) converges; then so does Σ[(uk + vk ) − uk ], but Σ[(uk + vk ) − uk ] = Σvk , so Σvk converges which contradicts the assumption that Σvk diverges. Suppose Σ(uk − vk ) converges; then so does Σ[uk − (uk − vk )] = Σvk which leads to the same contradiction as before. 28. Let uk = 2/k and vk = 1/k; then both Σ(uk + vk ) and Σ(uk − vk ) diverge; let uk = 1/k and vk = −1/k then Σ(uk + vk ) converges; let uk = vk = 1/k then Σ(uk − vk ) converges. ∞ ∞ 29. (a) diverges because (2/3)k−1 converges and 1/k diverges. k=1 k=1 ∞ ∞ (b) diverges because 1/(3k + 2) diverges and 1/k 3/2 converges. k=1 k=1 ∞ ∞ 1 1 30. (a) converges because both (Exercise 25) and converge. k(ln k)2 k2 k=2 k=2 +∞ +∞ 1 ke−k converges (Integral Test), and, by Exercise 25, 2 (b) diverges, because diverges k ln k k=2 k=2 ∞ ∞ ∞ 1 1 1 1 31. (a) 3 − = π 2 /2 − π 4 /90 (b) − 1 − 2 = π 2 /6 − 5/4 k2 k4 k 2 2 k=1 k=1 k=1 ∞ ∞ 1 1 (c) = = π 4 /90 (k − 1)4 k4 k=2 k=1 ∞ n ∞ 32. (a) If S = uk and sn = uk , then S − sn = uk . Interpret uk , k = n + 1, n + 2, . . ., as k=1 k=1 k=n+1 the areas of inscribed or circumscribed rectangles with height uk and base of length one for the curve y = f (x) to obtain the result. n (b) Add sn = uk to each term in the conclusion of Part (a) to get the desired result: k=1 +∞ +∞ +∞ sn + f (x) dx < uk < sn + f (x) dx n+1 k=1 n +∞ +∞ 1 1 1 33. (a) In Exercise 32 above let f (x) = . Then f (x) dx = − = ; x2 n x n n use this result and the same result with n + 1 replacing n to obtain the desired result.
  • 15. January 31, 2005 12:53 L24-ch10 Sheet number 15 Page number 438 black 438 Chapter 10 1 1 1 (b) s3 = 1 + 1/4 + 1/9 = 49/36; 58/36 = s3 + < π 2 < s3 + = 61/36 4 6 3 1 2 (d) 1/11 < π − s10 < 1/10 6 34. Apply Exercise 32 in each case: +∞ ∞ 1 1 1 1 1 (a) f (x) = , f (x) dx = , so < − s10 < (2x + 1)2 n 2(2n + 1) 46 (2k + 1)2 42 k=1 +∞ 1 π (b) f (x) = , f (x) dx = − tan−1 (n), so k2 + 1 n 2 ∞ 1 π/2 − tan−1 (11) < − s10 < π/2 − tan−1 (10) k2 +1 k=1 +∞ ∞ x k (c) f (x) = , f (x) dx = (n + 1)e−n , so 12e−11 < − s10 < 11e−10 ex n ek k=1 +∞ 1 1 35. (a) dx = 2 ; use Exercise 32(b) sw n x3 2n 1 1 (b) − < 0.01 for n = 5. 2n2 2(n + 1)2 (c) From Part (a) with n = 5 obtain 1.200 < S < 1.206, so S ≈ 1.203. +∞ 1 1 1 1 36. (a) dx = 3 ; choose n so that − < 0.005, n = 4; S ≈ 1.08 n x4 3n 3n3 3(n + 1)3 n n+1 1 1 1 37. (a) Let F (x) = , then dx = ln n and dx = ln(n + 1), u1 = 1 so x 1 x 1 x ln(n + 1) < sn < 1 + ln n. (b) ln(1, 000, 001) < s1,000,000 < 1 + ln(1, 000, 000), 13 < s1,000,000 < 15 (c) s109 < 1 + ln 109 = 1 + 9 ln 10 < 22 (d) sn > ln(n + 1) ≥ 100, n ≥ e100 − 1 ≈ 2.688 × 1043 ; n = 2.69 × 1043 38. p-series with p = ln a; convergence for p > 1, a > e 39. x2 e−x is decreasing and positive for x > 2 so the Integral Test applies: ∞ ∞ x2 e−x dx = −(x2 + 2x + 2)e−x = 5e−1 so the series converges. 1 1 40. (a) f (x) = 1/(x3 + 1) is decreasing and continuous on the interval [1, +∞], so the Integral Test applies. (c) n 10 20 30 40 50 sn 0.681980 0.685314 0.685966 0.686199 0.686307 n 60 70 80 90 100 sn 0.686367 0.686403 0.686426 0.686442 0.686454
  • 16. January 31, 2005 12:53 L24-ch10 Sheet number 16 Page number 439 black Exercise Set 10.5 439 √ √ +∞ 1 3 1 n3 + 1 3 2n − 1 (e) Set g(n) = dx = π + ln − tan−1 √ ; for n ≥ 13, n x3 + 1 6 6 (n + 1)3 3 3 g(n) − g(n + 1) ≤ 0.0005; s13 + (g(13) + g(14))/2 ≈ 0.6865, so the sum ≈ 0.6865 to three decimal places. EXERCISE SET 10.5 ∞ 1 1 1 1 1. (a) ≤ 2 = 2, converges 5k 2 − k 5k − k 2 4k 4k 2 k=1 ∞ 3 3 (b) > , 3/k diverges k − 1/4 k k=1 ∞ ∞ k+1 k 1 2 2 2 2. (a) 2−k > 2 = , 1/k diverges (b) 4+k < 4, converges k k k k k k4 k=2 k=1 ∞ ∞ 1 1 1 5 sin2 k 5 5 3. (a) < k, converges (b) < , converges 3k + 5 3 3k k! k! k! k=1 k=1 ∞ ln k 1 1 4. (a) > for k ≥ 3, diverges k k k k=1 ∞ k k 1 1 (b) > 3/2 = √ ; √ diverges k 3/2 − 1/2 k k k=1 k ∞ 4k 7 − 2k 6 + 6k 5 5. compare with the convergent series 1/k 5 , ρ = lim = 1/2, converges k→+∞ 8k 7 + k − 8 k=1 ∞ k 6. compare with the divergent series 1/k, ρ = lim = 1/9, diverges k→+∞ 9k + 6 k=1 ∞ 3k 7. compare with the convergent series 5/3k , ρ = lim = 1, converges k→+∞ 3k + 1 k=1 ∞ k 2 (k + 3) 8. compare with the divergent series 1/k, ρ = lim = 1, diverges k→+∞ (k + 1)(k + 2)(k + 5) k=1 ∞ 1 9. compare with the divergent series , k=1 k 2/3 k 2/3 1 ρ = lim = lim = 1/2, diverges k→+∞ (8k 2 − 3k) 1/3 k→+∞ (8 − 3/k)1/3 ∞ 10. compare with the convergent series 1/k 17 , k=1 k 17 1 ρ = lim 17 = lim = 1/217 , converges k→+∞ (2k + 3) k→+∞ (2 + 3/k)17
  • 17. January 31, 2005 12:53 L24-ch10 Sheet number 17 Page number 440 black 440 Chapter 10 3k+1 /(k + 1)! 3 11. ρ = lim = lim = 0, the series converges k→+∞ 3k /k! k→+∞ k + 1 4k+1 /(k + 1)2 4k 2 12. ρ = lim = lim = 4, the series diverges k→+∞ 4k /k 2 k→+∞ (k + 1)2 k 13. ρ = lim = 1, the result is inconclusive k→+∞ k + 1 (k + 1)(1/2)k+1 k+1 14. ρ = lim = lim = 1/2, the series converges k→+∞ k(1/2)k k→+∞ 2k (k + 1)!/(k + 1)3 k3 15. ρ = lim = lim = +∞, the series diverges k→+∞ k!/k 3 k→+∞ (k + 1)2 (k + 1)/[(k + 1)2 + 1] (k + 1)(k 2 + 1) 16. ρ = lim = lim = 1, the result is inconclusive. k→+∞ k/(k 2 + 1) k→+∞ k(k 2 + 2k + 2) 3k + 2 17. ρ = lim = 3/2, the series diverges k→+∞ 2k − 1 18. ρ = lim k/100 = +∞, the series diverges k→+∞ k 1/k 19. ρ = lim = 1/5, the series converges k→+∞ 5 20. ρ = lim (1 − e−k ) = 1, the result is inconclusive k→+∞ 21. Ratio Test, ρ = lim 7/(k + 1) = 0, converges k→+∞ ∞ 22. Limit Comparison Test, compare with the divergent series 1/k k=1 (k + 1)2 23. Ratio Test, ρ = lim = 1/5, converges k→+∞ 5k 2 24. Ratio Test, ρ = lim (10/3)(k + 1) = +∞, diverges k→+∞ 25. Ratio Test, ρ = lim e−1 (k + 1)50 /k 50 = e−1 < 1, converges k→+∞ ∞ 26. Limit Comparison Test, compare with the divergent series 1/k k=1 ∞ k3 27. Limit Comparison Test, compare with the convergent series 1/k 5/2 , ρ = lim = 1, k→+∞ k 3 + 1 converges k=1 ∞ ∞ 4 4 4 4 28. kk < k , kk converges (Ratio Test) so converges by the Comparison Test 2+3 3 k 3 2 + k3k k=1 k=1